You are on page 1of 78

Negligence v.

Strict Liability
Negligence failure to use ordinary care (reasonable person) Strict Liability assigns liability to category of actors regardless of fault Hammontree v. Jenner Epileptic, with knowledge of condition, crashes into store during seizure while driving. - The Court rejected Ps strict liability (prod liability) argument because to create strict liability for use of streets and highways creates too much uncertainty. It would require an entire infrastructure of regulations on top of what we already have. - Negligence, not strict liability will apply when a driver has knowledge of a potentially dangerous condition but not when it will occur, has been cleared to drive despite condition, causes injuries due to that condition Holmes Liability is to prevent evil acts. Evil acts cannot be prevented if unforeseen. Posner When the costs of prevention exceed the costs of expected damages, we cannot blame the defendant for negligence.

Vicarious Liability
Policy Assure compensation, prevent injury, equitable spread of losses, incentivize good hiring/sup General Rule Principal is not subject to liability for agent action EXCEPT when: (1) agent acts with apparent authority (and acts tortuously or in a manner that, if done by the principal, would be liable in tort), or (2) principal is negligent in selecting, hiring, supervising, etc., or (3) principal delegates performance of duty to use care to protect others arising out of some relation to public or plaintiff, or (4) the work is inherently dangerous Respondeat superior Principal is subject to vicarious liability when (1) agent is an employee acting within scope of employment, or (2) agent is acting under actual or apparent authority (both issues of fact). - Scope of Employment: 2d ROT of Agency 228 Christensen v. Swenson: Employee causes car accident while driving back from her lunch break. Three part Birkner standard for determining if employee is acting within scope of employment so that her employer is vicariously liable for her negligence under respondeat superior. 1. Was his conduct of the general kind the employee was hired to perform?

2. Was it substantially within the hours and boundaries of the employment? 3. Was it motivated, at least in part, by service to the employer's interest? Analysis: Vicarious liability gives employers incentives to cautiously select employees, then carefully supervise them and discipline them for being negligent. It assists in case of unintentional and intentional negligence. Doctrine of Apparent Authority Apparent authority exists where (1) Representation of authority by the principal. (2) Reliance on that principal by a third party. (3) A change in position by the third party based upon that representation. (2d ROT of Agency 267) (2d ROT 429 requires reasonable belief that services being provided by employer) Independent Contractors Not liable for acts of independent contractors unless ($. 429) services are accepted under reasonable belief they are being provided by the employer - 3d ROT 55 liability in negligence to those who hire IC: liable when activity IC is hired to perform creates a risk of physical harm and actors negligence is a factual cause of such harm within scope of liability, subject to 56 duty limitation: actor owes a duty of care to work entrusted in IC that they have retained control over 58-65: duty for particularly risky activities, ones regulated by statutes, etc Roessler v Novak: P received medical treatment, including

radiology work at D hospital, and radiologist made mistake by only making partial diagnosis. D argues radiologist is an independent contractor and they arent liable. - A principal may be held liable for the acts of its agent
within the course and scope of agency. Apparent agency is a question of fact and exists if: 1. The principal represents that there is agency 2. A third party relies on that representation 3. That reliance produces a change in position Concurrence: We want predictable rules, and efficiency; maybe it's time simply to specify in statute whether hospitals are vicariously liable, so we don't have to keep re-doing all of this. Analysis: Baptist Hosptial v Sampson the hospital had signs stating certain Drs were not employees and required patients to sign forms acknowledging hospital did not control their actions. P denied seeing these signs or signing the form and summary judgment was granted in favor of D. - Detrimental reliance: apparent agency, reliance on this, and change of position Direct v. Vicarious Liability

Principal Vicarious Liability Agent is negligent, but (for all the reasons listed above), the principal is vicariously liable. Principal Direct Liability In addition to being vicariously liable for the agents actions, the principal may be directly liable for their own negligent conduct (e.g. failing to perform background checks, instructing the agents conduct)

Negligence Principle
A. Definitions Negligence (2d ROT 282) conduct which falls below the standard established by law for the protection of others against unreasonable risk of harm Standard of Care (2d ROT 283) standard of conduct is that of a reasonable man under the consequences (unless actor is a child). Those ill or physically disabled held to standard of those in similar condition. NOT mental illness. o Children generally held to standard that accounts for age, etc. o 289 require defendant to exercise whatever superior attributes he possesses o 291 Where an act is one a reasonable man would recognize involves a risk of harm to another, the risk is unreasonable and the act is negligent if the risk is so great as to outweigh the utility of the act

B. The Standard of Care / The Reasonable Person Standard of care required is ordinary care such as is required by the exigencies of the case

Brown v. Kendall: One man hits another man in the eye with a stick by mistake while trying to break up a fight between their dogs. If, in the prosecution of a lawful act, a casualty purely accidental arises, no action can be supported for an injury arising therefrom. If both P and D were/were not using ordinary care, or if D was using ordinary care and P was not, then plaintiff cannot recover. Burden of proof is always on P to prove that the D failed to exercise reasonable care Ordinary care varies with circumstances: is that kind and degree of care that prudent and cautious men would use. The burden of proving a lack of care rests on P Analysis: Large impact on establishing fault principle, which was also established in Harvey v Dunlap where the misfortune of the sufferer versus the fault of the D is established. - Possibility that Shaw was motivated by making risk creating enterprises less hazardous to investors and entrepreneurs Where injury is unforeseeable and cost of avoidance high, the court should not impose liability. - Adams v. Bullock: Boy electrocuted when wire he is playing with touches trolley car wire. Cardozo: Negligence cannot be attributed to D because they used an overhead wire system rather than another. The trolley company exhibited effort to take reasonable precautions against injury by placing the wire out of reach and having no reason to predict such an incident as this could have occurred. Analysis: Events to not decide negligence, rather relative precautions do. Greene v Sibley: P was waiting for change after making a purchase in the store and she stumbled over the foot of a mechanic working on a register. - Cardozo wrote for 5 member majority, reversing, that the mechanic did not act negligently Objective Standard/Community Expectations The chief advantage of this standard of the reasonable (person) is that it enables the triers of fact ... to look to a community standard rather than an individual one. (2d ROT 283) - based on average community expectation (not average community behavior) The Reasonable Person Basic standard of care is measured against the reasonable, ordinary, prudent person. This person has similar physical characteristics, average mental ability, and same knowledge as average member of the community, measured from an objective standard.

One who exercises appropriate restraint in light of the interests of others - Neither the typical person, nor the rational person - Rejects subjective/personal standard (based on individuals actual ability) - Fault defined by objective standard, even though lay definition is based upon individuals actual abilities Negligence Equation (Learned Hand) B < P L = Negligence to maximize social benefit. The calculation of reasonable care without any consideration of morality. - U.S. v. Carrol Towing Co.: D's bargeman went ashore, and didnt oversee the "drill out" of a ship which came unmoored, crashed and sank with gov-owned cargo. Learned Hand: Liability arises when the product of the amount of damage multiplied by the likelihood of damage is greater than the cost of the precaution that would prevent an accident. Loss can by loss to society. - Since the cost of preventing the damage was low, and the likelihood of damage and the cost of the damage were high, D is liable. - (1) probability of injury, (2) gravity of injury, (3) burden of adequate precautions Posner Criticism: Does the reasonable person act like a rational profit maximizing enterprise? The rule cannot be applied to physical injury or loss of human life because they are incommensurate. Considers equation only in terms of liability on claims. Losee v. Buchanan (1873) exploding boiler. To be a part of society, you have to give up some natural rights. If as a civilization we want industry, then there are just some risks that we will have to tolerate. The owner was maintaining the boiler reasonably well so we cant make him liable for damages caused by the explosion (at the time of this case, lack of privity barred plaintiff from filing suit directly against manufacturer) Foreseeability Standard in determining negligence (3d ROT 3) An act is negligent based upon consideration of the following factors: (1) foreseeable likelihood of harm, (2) foreseeable severity of injury, and (3) burden that would be borne by the person and others if the person takes precautions that eliminate or reduce the possibility of harm. Remoteness of Possibility - Bolton v. Stone (GB 1951, Lord Reid) woman hit by exceptionally long cricket hit - Court considers remoteness of possibilities, seriousness of consequences, finds the test is whether a reasonable man would have taken further precautions in the circumstances -

Ordinary v Extra-Ordinary Care Separate standards no longer necessary - Common Carriers historically subject to standard of extraordinary care which effectively made them strictly liable Hegeman v. Western Railway (1855) Carrier is bound to use all precautions, as far as human care and foresight will go, for the safety of his passengers Bethel v. NYC Transit Authority bus wheelchair seat broke, D contends they should have been vigilant with inspection Cardozo Special duty of common carriers is inconsistent with fundamental concept of uniform standard of care Common carriers duty of highest care adopted during advent of steam railroad when primitive safety features resulted in an explosive growth in accident injuries that greatly affected tort system. Now safer and unnecessary. Example Jury Instructions re the Standard of Care - NY Pattern Jury Instruction 2:10 Negligence is failure to use that degree of care that a reasonably prudent person would have used under the circumstances - NJ Model Civil Charge 5.10 Adds the reasonable person is not the most cautious or bold but of reasonable prudence, negligence does not require bad intent - CA Adv. Comm. On Jury Instr. 401 Person is negligent if he or she acts how a reasonably careful person would not under the circumstances Children (402), disabled (403) are held to standard of similar people, intoxicated persons (404) must act as reasonable sober person would Dangerous Instrumentality-Wood v. Groh: P sues D whose 15 year old son shot him after breaking into a locked cabinet where his father kept a gun and ammo. Trial court enters judgment for D after ordinary negligence jury charge. Reversed and remanded on appeal where court holds instruction of highest care required because a gun is a dangerous instrumentality. Handicap: Res 1st 238B comments unless the actor is a child, his insanity or other mental deficiency does not relieve the actor from liability for conduct which does not conform to the standard of a reasonable man under like circumstances. Reasoning is that 1) it is too difficult to draw a line between mental deficiency and other variations of temperament, 2) ease of feigning mental deficiency, 3) mentally handicapped should have to compensate victims of their negligence as compared to keeping their wealth, 4) it incentivizes supervision of the mentally handicapped by their guardians.

Restatment 2d 238 C comments that standard of care for actor that is ill or otherwise physically disabled is reasonable man under like disability. 238C does not apply to mental disability. Superior attributes-Res 2d 289(b) requires defendant to exercise superior attributes as the actor himself has. Children-Standard is reasonable conduct for persons of their actual age, intelligence, and experience (Ellis v. DAngelo and Mastland Inc. v. Evans Furniture. Some states hold children under age 6 or 7 cannot be held negligent because of their inability to comprehend risk. These states use a rebuttable presumption for children between 7 and 14. When children engage in adult activities, courts apply adult standards (Dellwo v. Pearson 12 year old driving a motor boat).

C. The Role of Statutes


Definition a statute isnt just anything passed by the legislature but rather a law of general application (distinct from agency-created regulation, which can also be subject to judicial review, but courts are typically deferential to agency expertise) Huber Safety and the second best: the hazards of public risk management. Says courts should defer to safety standards put out by govt regulators who are better equipped to calculate and manage risks Uses of statutes creating or implying causes of action, establishing duty of care, standard of care (i.e. reasonable care or strict liability), specifying reasonable conduct Federal Law Eerie Doctrine (there is no such thing as federal common law) U.S. District courts are not courts of general jurisdiction so they cannot creates new causes of action (only congress can give federal courts new causes of action, but fed courts can expand application of statutes) Creating causes of action (2d ROT 874) - When statute requires / proscribes certain conduct but does not create a remedy, the courts can recognize a cause of action for redress to further the purpose of the legislation Statutory Violations as negligence per se (3d ROT 14) Actor is negligent if, without excuse, violates statute designed to protect against type of accident actor caused and accident victim is within class of persons statute is designed to protect. In NY, negligence per se applies only to state statutes, not municipal or county (though the latter may be considered as evidence).

Martin v. Herzog (NY 1920) buggy driver without lights, violating statute requiring lights at night, was hit by oncoming car. Cardozo says that violation of the statute is per se negligence and so obvious that reasonable minds could not differ (decided as a matter of law). Statute outlines duty of care required for safety of others, failing to follow statute is breach of duty Clinkscales v Carter (cal 1943) Not criminally liable for running stop sign, because it never became effective by ordinance. However, civil standard should rest with the court and common law tort that liability is based on can be found by a jury to have liability, in the absence of criminal liability. Statutory Violations are NOT negligence per se (3rd ROT 15) Violation is reasonable in light of actors childhood, physical disability or incapacity. Examples: Actor exercises reasonable care but sudden emergency renders violation of statute non-wrongful; Actor neither knows nor should know the factual circumstances rendering statute applicable; Violation due to confusing way requirements are presented to public; Compliance would create greater risk of harm to others than non-compliance Tedla v. Ellman (NY 1939) Two siblings are walking with traffic, rather than against per statutory requirement Where the statutes purpose is to do something other than safeguard people or prevent hazards, then whether one obeys it is not a standard for determining negligence. Rather, the standard is that of what a prudent man would do under the circumstances. Where the statutes purpose is to prevent harm, but following it would create a greater risk of the same harm, then you still dont have to follow it Example Jury Instructions re Statutory Violations NY PJI 2:26 (re traffic violations) In considering evidence, must determine whether plaintiff has proved the defendant failed to comply with a statute. If they violated, such a violation constitutes negligence. You cannot disregard violation of statute and substitute standard of care other than that in statute. 2:27 (justifiable noncompliance, to be read with 2:26) persons are also supposed to use common sense and reasonable care under the circumstances. Compliance may entail greater risk than noncompliance. If you find this is the case, the defendant is not chargeable with negligence.

D. The Roles of Judges and Juries


Judge Decide scope duty of care Jury What does reasonable care look like in the circumstance? 3d ROT 8 - when reasonable minds can differ as to the facts or as to whether the conduct lacks reasonable care, the function of jury is to determine those facts Attorneys Opinions do not matter NY PJI 1.6 Jury is sole judge of the facts, they are not to consider opinions of lawyers or judges as to the truth of the facts Historic view placed more emphasis on judge. Jury just assisted. Holmes: Court aids its conscience by taking the opinion of the jury. 3d ROT 37 Courts can determine as matter of law if there is insufficient evidence of an element or no reasonable jury could make a required finding. - This is incursion into jury area, different from determining legal rule to apply to class of cases Baltimore & Ohio Railroad v. Goodman (US 1927) categorical rule for standard of care regarding railroad crossings: unless you get out to look down the track, you are being negligent. - Did not send it to the jury because Holmes thought reasonable minds could not differ. - Holmess ruling essentially meant that railroad companies were never negligent - One could argue it is unreasonable to undertake Holmes proposed standard of care, the railway should have known about the danger of the crossing and undertaken additional precautions Pokora v. Wabash Railroad Co. (US 1934) limits Goodman to its facts. Its dangerous for courts to create specific standards of care outside of the reasonable person standard. Courts should be cautious in framing standards of behavior that amount to rules of law. He had a duty to be cautious, but there's a standard of prudent conduct at issue. Maybe stopping and getting out would be at least as dangerous as proceeding Since facts and standards always change, like technological factors, affecting care judges should refrain from rule-making where a jury would best determine the facts Judges do not have expertise in anything but the law (not, e.g., baseball fields). Atkins v Glen Falls School district (NY 1981): Majority held the school exercised reasonable care to P who was hit in eye with ball, dissent for the jury to say

Standards of prudent conduct are declared at times by the court, but they are taken over from the facts of life (sometimes it is clear how things are done, and they are done this way for a reason) Andrews v. United Airlines (9th Cir. 1994) woman is harmed by falling bag from overhead comp. Claim stated that UA knew of risk because they warned about it at the end of every flight. Court finds installation of safety nets is practical and not cost prohibitive, and a reasonable jury could conclude they did not act reasonably to prevent harm based upon the common carrier standard in California. Court sustains high degree of care required by California law

E. The Role of Custom


General Rule (3d Rest. 13b): An actors departure from the custom of the community, or of others in like circumstances, in a way that increases risk is evidence of the actors negligence but does not require a finding of negligence per se. Compliance (noncompliance) with custom is evidence the jury can consider, but does not preclude (require) a finding of negligence Custom Defined A widely established or observed pattern of conduct by a person or entity other than the actor who commonly engage in the same type of activity as the actor What custom is not A practice by strangers to the relevant community; Evidence of the defendants internal rules or practices is not evidence of custom (but a defendants safety protocols may establish a minimum standard of care based on the partys undertaking and the resulting expectations). Uses of Custom Offensive: to establish a standard of care from which the defendant has departed Defensive: to establish the reasonableness of the conduct To establish an element of the claim (e.g. foreseeability of an event) Rationale Customary practices invites reliance (keep to the right); Professional or industry standards may express accumulated successful experience; Custom may show recognition of danger to others Holmes what is customarily done is not necessarily the same as ordinary care. What usually is done may be evidence of what ought to be done, but what ought to be done is fixed by a standard of reasonable prudence, whether it is usually complied with or not.

10

Rule Custom can inform the standard of care when proof of a common practice aids in formulating the general expectation of society as to how individuals will act in the course of their undertakings, and thus to guide the common sense or expert intuition of a jury or commission when called on to judge particular conduct under particular circumstances (CB p.70). Trimarico v. Klein (NY 1982) - landlord fails to retrofit regular glass shower door with safety glass like hotels have retrofitted. If the action is not customary in the industry in question, it does not apply. While it was customary for hotels to do so, it was not a requirement the landlord do the same. The existence of custom is not enough to establish negligence, BUT its existence is enough to raise question of fact for the jury. Holmes: what usually is done is not necessarily what ought to be done Levine v Blaine Co : custom to equip dumbwaiter with rope that doesnt splinter Converse Rule Custom is not a complete defense. Morris v. West Hartlepool Steam Navigation (House of Lords 1956) Plaintiff was injured when, for want of a guard railing, he fell into the ships hatch during an errand while out at sea. Simply because something is a custom does not mean that it reflects the standard of ordinary care. There may be times where custom should be improved/changed. Compare two simultaneous customs of ship-owners: (1) They typically have no guard rails up while out at sea, and (2) They almost always have guard rails are up while at port to comply with the Port of Authoritys requirements. The former custom, in consideration of the latter custom, makes it lower than a standard of reasonable care. Rule on lack of Custom Just because it is customary not to take some action (i.e. custom does not provide for positive action), that doesnt mean the omission is not negligent. TJ Hooper Case (2d Cir. 1932) tug captains customarily do not have radios, are unaware of rough weather which sinks two coal barges. Industry failures require judicial intervention typically we dont believe courts have the expertise to decide what should be custom in an industry, but the industry here failed to self-patrol. Courts must in the end say what is required; there are precautions so imperative that even their universal disregard will not excuse their omission. Modern Law Due to industry inertia, courts have rejected the argument that a prevailing custom defines the standard of care (except in malpractice cases)

11

F. Proof of Negligence
Terms of Art - Burden of Proof Burden of Persuasion: Always on P to prove by a preponderance of evidence. Burden of Production: Typically shifts to D to prove he was not acting negligently after P has brought forth a prima facie case - Typically, D has greater access to evidence so it shifts Notice & Knowledge Actual knowledge (someone tells you); constructive knowledge (based on reasonable inference); imputed knowledge (manufacturers are expected to stay current in their field); special knowledge (trade practices, standards, industry knowledge) Constructive Notice General Rule Defendant must have had sufficient time to discover and remedy the dangerous situation to be found liable, general awareness of potential hazard is not sufficient to prove constructive notice Negri v. Stop & Shop (NY 1985) Slip and fall in a grocery store because broken jars of baby food were not timely cleaned up. The evidence showed that the spill had not been cleaned up for several hours. - Circumstantial Evidence Lower courts dismissed because Ps only evidence was circumstantial. Court of Appeals reversed, holding that as a matter of law, the jury could have sufficient evidence (even if just circumstantial) to infer that D acted negligently. - Constructive Notice due to the amount of time that had elapsed, D had constructive notice of the dangerous condition and, therefore, also enough time to clean it up. Gordon v. Am. Mus. Nat. Hist. (NY 1986) Slip and fall outside the museum on the front steps caused by wax paper from the vender, whom the museum contracted to be located in the entranceway of the building. - Defect must be visible and apparent for sufficient length of time to permit discovery and remedy - No Constructive Notice if knowledge is of general, transient hazard unlike the spilled baby food that had been there for a while, or a case where the condition of the stairs had fallen into disrepair, the specific litter causing plaintiffs injury was transient and general knowledge that litter could and does occasionally exist is not enough to demonstrate constructive knowledge. Kelly v Stop and Shop (CN 2007) P slipped on lettuce at selfserve salad bar. Adopted business practice/mode of operation rule - customer need not establish actual or constructive notice when business practice of store provided a continuous and foreseeable risk of harm to customers

12

Circumstantial v Direct Evidence neither is required Circumstantial evidence permits you, if you choose to, to draw an conclusion from one fact that another exists (most evidence is circumstantial, e.g. theres white stuff on the ground outside, so it must have snowed last night) Res Ipsa Loquitur (The Thing Speaks for Itself) using circumstantial evidence 2d ROT 328 Negligence can be inferred when event is of kind that ordinarily does not happen without negligence, other causes are sufficiently excluded, and negligence is within scope of duty to plaintiff 3d ROT 17 Fact finder may infer negligence when accident causing harm is of type that ordinarily happens as a result of negligence and defendant is member of class of actors Defined (quoted from McDougald): Essentially, the injured plaintiff must establish that the instrumentality causing his or her injury was under the exclusive control of the defendant, and that the accident is one that would not, in the ordinary course of events, have occurred without negligence, on the part of the one in control (CB p. 96). - Elements (excerpted from Ybarra): - The accident must be of a kind, which normally does not occur in the absence of someones negligence. - It must be caused by an agency or instrumentality within the exclusive control of the defendant. - It must not have been due to any voluntary action or contribution on the part of the plaintiff. Not strict liability you still have to prove negligence Judge determines whether the inference can be reasonably drawn, jury determines whether the inference should be drawn Ordinarily does not occur absent negligence is a fact and can be proven by: - Common knowledge - Judicial knowledge (when judge makes decision as a matter of law) - Expert testimony Burden Shifting once you establish res ipsa, then the burden shifts to the defense because theyre the only ones with access to the evidence. - D must overcome the reasonable inference that the accident would not have occurred without negligence. - BUT jury does not have to find for the plaintiff, even if D doesnt present any contrary evidence. Though there is a prima facie case, that does not mean it was conclusive (otherwise, it would have been a judgment as a matter of law)

13

Byrne v. Boadle (1863) p 92 barrel of flour falls out window, hits man. Res ipsa relieves P of burden to prove exact circumstances of accident. Spoilation of Evidence Spoliation of evidence (both intentional and negligent destruction) courts largely ignore to avoid endless litigation (p 101). McDougald v. Perry (FL 1998) p 95 tire falls off trailer and injures plaintiff driving behind. Plaintiff is not required to eliminate with certainty all other possible causes or inferences is common sense inference Res Ispa permits, but does not compel an inference of negligence, as it is merely a rule of evidence (Marrero v Goldsmith 486 So.2d 530) - Goodyear tire v Hughes supply: provides injured P with common sense inference of negligence where direct proof is wanting, provided certain elements consistent with negligent behavior are present - Instrumentality causing P injury was in exclusive control of D and accident would not have happened in ordinary events without negligence At the prima facie phase, all that P has to do is produce reasonable evidence of negligence (e.g. with res ipsa) and then the trier of fact can decide if it meets the burden of proof later on (where that evidence can be presented) Evidence inaccessible to plaintiff When P is unable to access evidence for example due to unconsciousness, D has burden to produce evidence which is practically accessible to the D but inaccessible to the P. - Ybarra v. Spangard (CA 1944) p 102 P suffers nerve damage in arm after appendectomy. P could not determine instrumentality of injury nor the specific Ds, so he sued everyone in the op. room at the time. Today Since then, many courts (Barrett v Emanuel Hospital) have rejected Ybarra saying that a modern discovery practices casts doubt on the need to apply res ipsa loquitur so broadly. Also, special protections for unconscious patients, specifically, can be achieved through strict liability (as the judge in Ybarra alluded to) and/or respondent superior liability.

14

Chin v. St. Barnabas Medical Center (NJ 1999) Woman dies due to clear negligence of pumping gas not fluid into her body. When plaintiff receives unusual injuries while unconscious, all defendants who had any control over her body may properly be called upon to meet the inference of negligence by giving explanation of their conduct, and the burden of proof shifts to them, since she clearly was blameless.

G. Medical Malpractice The Standard of Care for Professionals


Elements (in NY): o Duty: physician-patient relationship (does not require payment, can be free service) o Breach: negligence in diagnosis/treatment of condition for which patient is under care of physician o Causation: negligence substantial factor in causing the plaintiff to be injured This is only for negligence! Not misconduct. The practitioner is not being criminally prosecuted; his license is not in jeopardy. Locality Rule (no longer relevant) The doctrine that, in a professional-malpractice suit, the standard of care to be applied to the professional's conduct is the reasonable care exercised by similar professionals in the same vicinity and professional community. Defined: [W]hen a physician undertakes to treat or diagnose a patient, he or she is under a duty to exercise the same degree of diligence and skill which is commonly possessed by other members of the profession who are engaged in the same type of practice in similar localities having due regard for the state of scientific knowledge at the time of the treatment (p 113). Sheeley v. Memorial Hospital (RI 1998) p 111 Court declined to apply strict locality rule in evaluating expert witness, said it was irrelevant in the case at hand (botched C-section). Primary concern is whether treatment is administered in a reasonable manner The locality rule should be limited as it legitimizes a low standard of care in smaller communities. It should not be used to disqualify otherwise admissible testimony. The court basically replaced locality with class + circumstances. With modern transp and comm, local community standards is no longer relevant. Standard of Care for a prima facie claim, plaintiff must prove what the accepted standard of care was and that the doctor deviated from that standard

15

Custom In medical malpractice, custom is the standard of care. It has to be recognized as the standard of care (not just the personal opinion of one expert opinion. The expert is just relaying the information that the profession as a whole has established.) Good faith in determining treatment is irrelevant; what matters is whether they upheld the standard dictated by the profession (DiFranco v. Klein, RI 1995) BUT, use of judgment still relevant to choosing a course of treatment among several choices, all of which are accepted in the industry Superior Skills Providers must use their best judgment and whatever superior knowledge they possess, even if it exceeds the average provider in the area McCullogh v. U. of Rochester (App. Div. 2005) 3d ROT 12 - Superior skills and knowledge are circumstances to be considered in determining whether they acted as reasonable person Res Ispa Loquitur: used in cases where it is not clear what exactly caused injury, but all probable causes are within control or right of D Sides v. St. Anthony's Medical Center (MO 2008) P developed infection during surgery while unconscious and her body and the surgical site was under exclusive control of Hospital. Infection does not happen w/o negligence and D had greater knowledge of what happened Hasemeier v Smith (1962) D argued this prevented RIL claim in medical malpractice w/o specific negligence theory. Court distinguished facts in this case were insignificant but RIL could be used, with expert testimony to bridge gap and show it couldnt happen w/o negligence Proof of Breach Common Knowledge no expert testimony needed when negligence is readily apparent to anyone of average intelligence and ordinary experience Klimko v. Rose (NJ) chiropractor continues to apply pressure to neck after patient turns blue Chin v. St. Barnabas readily apparent that hoses were misconnected Expert Testimony provide insight into technically complex matters State v. Lourdes Hospital (NY 2003) p 119 P suffered injury to her arm from improper placement by her anesthesiologist. Res ipsa is not limited to common knowledge.

16

After establishing the other requirements of res ipsa loquitur (i.e. exclusive control and absence of contributory negligence), P may use expert testimony to bridge the gap of information the jury may face with technical cases. The jury, however, is still ultimately responsible for determining the validity of the facts. Admissibility of Evidence (judge as gatekeeper) Judge has the obligation to only allow testimony that is reached by the scientific method and employing the degree of rigor that is employed in the field and that it fits with the facts sought to be proved in the case. The judge may not allow a witness to testify (lay or expert) if the judge believes them to be baseless. Court must determine if opinions probative value outweighs prejudicial value Expert Testimony (1) Must be qualified, (2) Judge must ensure testimony is legitimate to go to jury, based on sufficient facts and data, (3) product of reliable principles or methods, (4) exercise degree of rigor usual in the field, and (5) professional, non-subjective opinion Fed. Rules of Evidence Rule 702 on Expert Testimony [testimony must be reliable and fit] If scientific, technical, or other specialized knowledge will assist the trier of fact to understand the evidence or to determine a fact in issue, a witness qualified as an expert by knowledge, skill, experience, training, or education, may testify thereto in the form of an opinion or otherwise, if (1) the testimony is based upon sufficient facts or data, (2) the testimony is the product of reliable principles and methods, and (3) the witness has applied the principles and methods reliably to the facts of the case. Doctrine of Informed Consent / Self Determination General tort law is a tension between liberty and security. Only applies when there is going to be some sort of invasion. No court has applied it where a Dr elected not to perform surgery New York CLS Pub Health 2805-d (2005) Limitation of Medical, Dental or Podiatric Malpractice Action Based on Lack of Informed Consent Definition: Lack of informed consent means the failure of the person providing the professional treatment or diagnosis to disclose to the patient such alternatives thereto and the reasonably foreseeable risks and benefits involved as a reasonable medical, dental or podiatric practitioner under similar circumstances would have disclosed, in a manner permitting the patient to make a knowledgeable evaluation. Limits: Lack of informed consent must be based upon either (a) non-emergency treatment, procedure or surgery, or (b) a diagnostic procedure involving invasion or disruption of the integrity of the body.

17

recognizes the inherent risks of even diagnostic measures. Objective Standard: The lack of information means that a reasonably prudent person in the plaintiffs situation would have refused care or chosen alternate care had s/he been informed. Lack of informed consent is the proximate cause of injury Defenses: 1. The risk was common knowledge 2. Patient chooses to be uninformed 3. Consent was not reasonably attainable 4. The medical professional deems informing the patient would have been directly harmful to his condition New Jersey Mathies v. Mastromonaco (NJ 1999) woman breaks hip, doctor does not seek consent to pursue nonsurgical method of treatment. Informed consent applies to non-invasive procedures not just invasive procedures. Self-determination rests with the patient. The doctor cannot impose his risk tolerance or values on the patient. Even if no physical injury results, there is psychological harm of not being able to decide own future. Reasonable standard switches from what doctors believe should be told to the patient to what a reasonable patient would want to know. Shine v Vega (Mass 1999) in life threatening situations, and when incapable of giving consent, consent is not needed and jury should determine if life threatening Sample Jury Instructions Michigan Purely statute, requires breach of standard of care, for specialists considers facilities reasonably available under the circumstances New York Negligence, looks at standard in relevant medical community, doctors must comply with (statewide, national, industry) minimum standards, must use any superior knowledge and skills they possess New Jersey Purely common law, GPs represent they have knowledge and skill normally possessed and used by average doctor practicing as a GP (NJ Model Jury Charge 5.36) NJ MJC 5.36 - Doctor must discuss all reasonable courses of treatment, including non-treatment, and probable risks and outcomes of each Information is material when reasonable patient would consider it important in deciding on treatment

18

Plaintiff must prove (1) failure to give information, (2) undisclosed risk occurred, (3) reasonable person would not have gone through treatment had risk been disclosed, and (4) course of treatment was proximate cause of injury

Ch. 3 Duty Requirement


Threshold determination but a flexible concept (Prosser 1971 30): Accordingly the standard of conduct is care commensurate with the reasonably foreseeable danger, such as would be reasonable in the light of the recognizable risk, for negligence is essentially "a matter of risk that is to say of recognizable danger of injury". Prosser Obligation recognized by law to conform to particular standard of conduct towards another Conk Duty determination is a determination of law by a court about a certain category of recurrent situations in which a court determines that such a person, when he or she acts, and puts another at risk, owes a duty of reasonable care unless the court determines that an immunity or privilege to act in such a way or the court, for policy reasons, which are generally exceptional, concludes the duty should not extend so far or should be modified.

19

Matter of Law Duty determinations are made by legislature or judicial determination Foreseeability The Restatement is criticized for not including foreseeability in its duty calculation (even though, historically, foreseeability is a major duty consideration). Owen for an unintentional negligence claim, it had to have been foreseeable. Owen predicts that the Restatement will have little impact on the law. Restatement does not include foreseeability is a response to the criticism that making foreseeability a part of the duty, then makes the judges intrude into the fact-finding realm of the jury. Neighbor Principle: Donoghue v. Stevenson (En. 1932) must take reasonable care to avoid acts/omissions which you can reasonably foresee and would likely injure your neighbor Rowland v. Christian Totality of the Circumstance Test Policy may call for departure from the general rule of no liability for conduct of other persons. Instead of working around inane categories (e.g. trespasser, licensee). Considerations: foreseeability of harm to the plaintiff the degree of certainty that the plaintiff suffered injury the closeness of the connection between the defendant's conduct and the injury suffered the moral blame attached to the defendant's conduct the policy of preventing future harm, the extent of the burden to the defendant and consequences to the community of imposing a duty to exercise care with resulting liability for breach, and the availability, cost, and prevalence of insurance for the risk involved.

A. Affirmative Duties
Duty (3d ROT 37): a) An actor ordinarily has a duty to exercise reasonable care when the actors conduct creates a risk of physical harm. b) In exceptional cases, when an articulated countervailing principle or policy warrants denying or limiting liability in a particular class of cases, a court may decide that the defendant has no duty or that the ordinary duty of reasonable care requires modification. No Duty of Care for Risks not Created by Actor (3d ROT 37): An actor whose conduct has not created a risk of physical harm to another has no duty of care unless a court determines that one of the affirmative duties provided in 38-44 is applicable.

20

Duty Created by Statute (3d ROT 38): When a statute requires an actor to act for the protection of another, the court may rely on the statute to decide that an affirmative duty exists and its scope. Duty by Risk-Creating Conduct (3d ROT 39): When an actor's prior conduct, even though not tortious, creates a continuing risk of physical harm of a type characteristic of the conduct, the actor has a duty to exercise reasonable care to prevent or minimize the harm Special Relationships / Affirmative Duties (3d ROT 40): a) An actor in a special relationship with another owes the other a duty of reasonable care with regard to risks that arise within the scope of the relationship. b) Special relationships giving rise to the duty provided in Subsection (a) include: a. a common carrier with its passengers, b. an innkeeper with its guests, c. a business or other possessor of land that holds its premises open to the public with those who are lawfully on the premises, d. an employer with its employees who are: i. in imminent danger; or ii. injured and thereby helpless, e. a school with its students, f. a landlord with its tenants, and g. a custodian with those in its custody, if i. the custodian is required by law to take custody or voluntarily takes custody of the other; and ii. the custodian has a superior ability to protect the other. No Duty to Rescue (2d ROT 314) Generally no duty to act for protection of others or to rescue Social Host Duty to Guests o Harper v. Herman (Minn. 1993) p 134 20 year old dives into shallow water off defendants boat, and is paralyzed. No relationship that created duty. D was a social host. To have had an affirmative duty to P, P must have been in a restricted or vulnerable positionone in which he was unable to self-protect. Alternatively, D would have needed to be in a position to gain economically from the relationship. Neither criterion was met; as a social host, D does not have an affirmative duty to act. No special relationship. Duty v Breach This ruling determined that Herman had no duty towards Harper. It does not establish that, if there were a duty, whether Hermans action/omission was not negligent.

21

Slippery Slope Court does not want to intrude too much into regular social activities; would rather put the burden on the risk-taker Duty to Another Based on Taking Charge of the Other (3d ROT 44) a) An actor who, despite no duty to do so, takes charge of another who reasonably appears to be (1) imperiled and (2) helpless or unable to protect himself or herself has a duty to exercise reasonable care while the other is within the actor's charge. b) An actor who discontinues aid or protection is subject to a duty of reasonable care to refrain from putting the other in a worse position than existed before the actor took charge of the other and, if the other reasonably appears to be in imminent peril of serious bodily harm at the time of termination, to exercise reasonable care with regard to the peril before terminating the rescue. Farwell v. Keaton (Mich. 1976) p 140 Guy gets beaten up, friend starts to take care of him, leaves him in backseat overnight and he dies. Rule: One who gratuitously acts for the benefits of another, although under no duty to do so in the first instance, is then under a duty to act like an ordinary, prudent, reasonable person and continue the assistance. Factual circumstances may give rise to a duty to act. In this case, friend had a responsibility not to make the situation worse through his inaction. Special relationship based upon two things: (1) companionship or joint venture but more importantly, (2) that the friend knew (and was arguably the only person who knew) of the peril the decedent was in. Deprived him of getting help from others, thus took him into his custody and owed him duty to help Social policy implications for ignoring someone elses peril, which the court, through this ruling, hopes to discourage. Moch v. Rensselaer Water Co. (NY 1928) p 146 RWC responsible for keeping adequate water in the fire hydrants, which Moch alleged was not adequate and led to fire spreading from its original location to their warehouse and burning it down. (1) Incidental and secondary beneficiaries of contracts do not have a claim. The property owners are secondary; the city itself is primary since they have the contract with RWC. (2) Omission of a benefit is not the same as an act (nonfeasance does not equal misfeasance). Making RWC liable would too greatly expand the zone of duty. The undertaking doctrine has largely overturned Cardozos ruling.

B. Duty Owed to a Third Party

22

Negligent Misrepresentation 2d ROT 311 Negligent Misrepresentation (1) One who negligently gives false information to another is subject to liability for physical harm caused by action taken by the other in reasonable reliance upon such information, where such harm results (a) to the other, or (b) to such third persons as the actor should expect to be put in peril by the action taken. (2) Such negligence may consist of failure to exercise reasonable care (a) in ascertaining the accuracy of the information, or (b) in the manner in which it is communicated." Randi W. v. Muroc Jt. Unified School District (CA 1997) p 148 Admin transferred districts with glowing recs even though he had history of sexual misconduct. While D had no duty to third party for negligence per se (because she was not in the protected class identified by the statute), D still had a duty re negligent affirmative misrepresentations (general rule is no duty regarding nondisclosure). 1st, D owed P a duty not to misrepresent the character of the vice principal, since it was reasonably foreseeable that an employer would read the unequivocally favorable letters and hire the vice principal and that he would molest a student such as plaintiff. 2nd, defendants' letters constituted misleading halftruths rather than mere nondisclosures. 3rd, as to reliance, although D made no representations to P, she was entitled to protection, since she suffered physical injury resulting from the reliance of the district that ultimately hired the vice principal. 4th, plaintiff adequately pleaded causation between defendants' misconduct and her injuries. Ballard v Uribe Determining duty is not to decide whether a particular Ps injury was foreseeable in light of a particular Ds conduct, but to evaluate generally whether the category of negligent conduct is likely to result in harm experienced, that liability may appropriately be imposed on negligent party Duty to Control the Acts of Another General rule: There is NO DUTY to control the acts of another 3d ROT 315 There is no duty so to control the conduct of a third person as to prevent him from causing bodily harm to another unless, a) a special relation exists between the actor and the third person which imposes a duty upon the actor to control the third person's conduct, or

23

b) a special relation exists between the actor and the other which gives to the other a right to protection. Exception to the rule: Tarasoff v. Board of Regents (CA 1976) p 157 P is the family of decedent, who was killed by patient of D psychologist, after the patient made known to him that he was going to murder the decedent. Holding: (1) Special relationship is sufficient to establish duty of care, even when the harm is suffered by a third party. (2) It is within the realm of reasonable care for psychologists to predict future harm and, thereby, warn patients. However, in this case the plaintiff alleged that the psychologist had already predicted harm (hence recommending the patient be detained), but failed to warn. Thus, the holding is regarding failing to warn, and the part about predicting harm is dicta. It wasnt enough that he called the police to detain the patient; he should have warned the potential victim. Public policy favoring protection of the confidential character of patient psychotherapist communications must yield to the extent to which the disclosure is essential to avert danger to others. The protective privilege ends where the public peril begins. Since this case, the CA legislature enacted a civil code which made psychologists immune from claims unless there was a serious threat of violence the victim was reasonably identifiable. Tenuto v Lederle Lab P contracted polio after coming in contact with vaccinated daughter. P fell within determinate class of immediate family members and special relationship provided for expanded duty of care Duty further supported in that Dr. was a pediatrician, involving interconnections of reliance between Ps and doctors

C. Policy Reasons for Invoking No Duty of Care


Statutes: public v private action Uhr v East Greenbush Central School District (NY 1999) School district fails to test young child for scoliosis. He gets it. Can the plaintiff's claim that the education law was violated be a cause of private action? - A private cause of action would be inconsistent with the legislative intent, basically. In order for a statute to create a private cause of action, three prongs must be satisfied: P must be a member of the class for whose benefit the statute was enacted A private right of action would promote the legislative purpose

24

The creation of such a right would be consistent with the legislative scheme - Here, we've got 1 and 2, but not 3, because another kind of enforcement (commissioner can withhold funding) is explicitly defined. Cuyler v US (2004) babysitter fatally abused baby, and personnel at hospital violated IL statute by failing to report suspicion of child abuse. No common law duty to report, concluded no private right of action should be found in the statute. Privity / Contractual Relationships Strauss v. Belle Realty (NY 1985) p 176 citywide blackout, old man falls in common area of apt, landlord has contract with ConEd for elect in comm. areas No liability because there is no privity between P and ConEd. Also, ConEd would be subject to limitless liability. Duty is defined neither by foreseeability nor by contract. While absence of privity does not foreclose duty, courts must fix the orbit to limit legal consequences of wrongs to a controllable degree. Dissent: The public policy argument is perversesaying that the more parties are potentially harmed by a tort-feasors gross negligence, the less responsibility they should have for injuries incurred. Spreading the risk. We should place the burden on whoever is in a better position to manage the risk: a huge utility company or an individual plaintiff? Social Host Duty to Third Parties Reynolds v. Hicks (WA 1998) p 183 Plaintiff was injured by a minor who was served alcohol by defendant host at their wedding reception. Courts reluctant to extend liability to social hosts (except NJ) Social hosts are not the same as commercial vendors. Social hosts are not as able to monitor and be responsible for their minor guestseven though the same is required of commercial vendors (and comm. venders have a duty to third parties harmed by minors who received alcohol at their establishments). Dissent: The majority is saying that social hosts who serve alcohol to minors (making them in violation of the criminal code) are not civilly liable to damages caused to third parties. Both social hosts and commercial vendors who supply alcohol to minors are committing criminal acts. Why distinguish between the two for civil causes of action? Negligent Entrustment

25

2d ROT 390 Elements (notice, res again refrains from using foreseeability): Supply chattel to another The supplier has reason to know that the recipient is somehow deficient (e.g. inexperienced) Recipients use of the chattel creates unreasonable risk Physical harm takes place Vince v. Wilson (VT 1989) p 188 family buys kid a car, he causes accident negligent entrustment requires a showing that the entrustor knew or should have known some reason why entrusting item to another was negligent or foolish. D. Foreseeability & Duty Determinations Paradox: Foreseeability is a factual consideration, and judges should not overstep into the responsibilities of juries. However, by not considering foreseeability in their duty determinations (which are made as a matter of law), then the case is unlikely to get to the jury in the first place. It will be dismissed for want of duty. Without the considering foreseeability, what then do you use to determine if there is a duty? Just blanket categorizations and policy implications? E.g. Yes or no, does the owner of a business owe a duty to customers on its property to prevent harm committed by third parties? The Asbestos Wives: Compare New York & New Jersey Holdampf v. A.C. & S., Inc. (In re New York City Asbestos Litigation) (NY 2005) Facts: laundry services included No Duty to Wife: The Port Authority of NY/NJ owes no duty of care to P wife, who was allegedly injured from exposure to asbestos dust that husband, a Port emp, introduced into home on soiled work clothes that plaintiff wife laundered. Did Port Authority create risk, and extension of duty, by allowing asbestos to be taken off premises? D argues that the employer owes no duty of care to non-employees. P argues that Port Authority was negligent in that it allowed asbestos to be taken off its property, which created a hazard for third parties (similar to cases involving mining nuisances). There are cases in other jurisdictions where the courts have ruled that there is a duty of care to third parties, but none in New York. Courts traditionally fix the duty point by balancing factors, including the reasonable expectations of parties and society generally, the proliferation of claims,

26

the likelihood of unlimited or insurer-like liability, disproportionate risk and reparation allocation, and public policies affecting the expansion or limitation of new channels of liability. Thus, in determining whether a duty exists, courts must be mindful of the precedential, and consequential, future effects of their rulings, and limit the legal consequences of wrongs to a controllable degree (numbers added). Foreseeability does not determine duty (it just helps to determine level of negligence once a duty has been established). Also, any expansion of the scope of duty should be weighed against the social costs. The court should not expand duty by one party (employer or landowner) over third parties that the employer or landowner has no way of controlling. There is no relationship between the Port and the plaintiff much less a special relationship (e.g. master and servant, parent and child). Ultimately, the NY court makes a slippery slope argument. Olivo v. Owens-Illinois, Inc. (NJ 2006) Facts: no laundry service Duty to wife: A landowner can be liable for injuries allegedly caused from asbestos exposure experienced by wife of a worker who performed fitting tasks that brought him into contact with asbestos on the landowner's premises. (1) Foreseeability Whether a duty is owed by a landowner to a person who is exposed to a hazard off-premises based upon a condition created on the landowners premises is a question of the foreseeability of the risk of harm to that individual or identifiable class of individuals. The defendant should have foreseen the risk to someone like the decedent who was an immediate family member of an employee who was likely to become exposed to the dangerous chemicals brought into the home from the worksite. (2) Balancing Factors Once the foreseeability of an injured party is established, the determination of whether imposing a duty is fair involves weighing and balancing several factors (1) the relationship of the parties, (2) the nature of the attendant risk, (3) the opportunity and ability to exercise care, and (4) the public interest in the proposed solution. There is no record that the defendant did anything to warn the workers or their families of the risks involved with handling clothing covered in asbestos dust, nor did they provide options for the workers to minimize the risk (e.g. by offering onsite laundry services).

27

(3) Fairness No need to fish out classifications of relationships (e.g. master-servant). Rather, simply examine the actual relationship present, and see if applying a general duty to exercise reasonable care in preventing foreseeable harm is fair and just. Once foreseeability determined, then whether or not duty is warranted is based upon policy considerations. Fears of a slippery slope are overstated. Technical note: even though the husband was an independent contractor (and not an employee) the landowner still has a duty to make the premises safe for invitees. The case was remanded to see if there was a duty to owed to the husband based upon the standards used to determine duty to independent contractor (if there is a duty to him, then there would be a duty to his wife).

E. Duties of Landowners and Occupiers


Three ways to analyze duty owed by landowners: Common Law (semantic morass) The Rowland Totality of the Circumstances Test (includes foreseeability) The Third Restatement (you owe a duty to everybody, but less to flagrant trespassers) Common Law Categories: The semantic morass Trespassers: non permissive entrants Licensees: permitted entrants A social guest is a licensee Invitees: invited entrants Prosser: make out a general sliding scale. As legal status of visitor improves, possessor owes a greater duty of protection Reflect foreseeability of injuries to others, obligation of injured person to foresee precautions likely to be taken for his benefit, varying economic burdens justifiably placed on possessor Carter v. Kinney (MO 1995) p 195 Bible study participants from defendants church slipped on ice and fell. Court held plaintiff was a licensee and was owed no duty with respect to unknown dangerous condition. - There was no material benefit at stake, no commercial relationship, and no evidence that the public at large was invited. Someone invited for purposes not strictly social is not necessarily an invitee. Social guests are a subclass of licensees. 2d ROT 329-31 Trespasser: one who enters on or remains on property w/o privilege or consent of owner

28

Possessor has duty to refrain only from inflicting willful, wanton or intentional injuries (2d ROR 333) Traps: (1) artificially created (2) inherently dangerous (3) but deceptively innocent instrumentality or condition Licensee: one who is permitted to enter or remain upon property with possessors consent, express or implied Refrain from inflicting willful, wanton, or intentional injury, or maintaining a trap Refrain from committing acts of affirmative negligence Exercise reasonable care to disclose dangerous defects known by possessor and unlikely to be discovered by licensee 341-2 Invitee: Public: invited as a member of public to enter or remain on prop which is held open to public Business: invited to remain on prop for purpose directly/indirectly connected with business dealings with possessor of property Use due care to keep prop in reasonably safe condition so they will not be unnecessarily exposed to danger 341a, 343, 343a Heins v. Webster County (NE 1996) p 201 Man either visiting daughter who works at hospital (licensee) or arranging volunteering as Santa (invitee) Court abolishes CL classifications of invitee and licensee and requires a duty of reasonable care for all non-trespassers. Argument in favor of keeping the categories: its more predictable so landowners can better allocate risk. Argument for dumping categories: a mans life or limb should not become less worthy of protection depending upon minute specs of what he was doing on the property; its no longer such a burden for landowners/possessors to exercise reasonable care with regards to all entrants on their property; status can be taken into consideration when determining foreseeability in general negligence principles; so many courts have already dumped it (most notably the Supreme Court in Kermarec) Duty v. Breach Heins is using the Rowland elements to determine whether or not there was breach (negligence) even though Rowland used it to decide duty. (Here, the duty element has already been satisfied regarding all non-trespassers): The foreseeability or possibility of harm The purpose for which the entrant entered the premises -

29

If the intent of the entry is unlawful (there can be other unlawful intentions beyond the entry but they dont matter), then thats a disqualifier. The time, manner, and circumstances under which the entrant entered the premises The use to which the premises are put or are expected to be put The reasonableness of the inspection, repair, or warning The opportunity and ease of repair or correction or giving of warning The burden of the land occupier and/or community in terms of inconvenience or cost in providing adequate protection Criminal Conduct by a Third Party Posecai v. Wal-Mart Stores (LA 1999) p 211 P robbed in Sams Club parking lot, sues store. Business owners have a duty to implement reasonable measures to protect their patrons from criminal acts when those acts are foreseeable, but in this case, they decided it wasnt foreseeable (history of too few incidents) so no duty. Various approaches to determining foreseeability: Specific harm rule owner must be aware of imminent danger. Too restrictive. Prior similar incidents test foreseeability established by evidence of previous incidents. Applied pretty arbitrarily though (how frequent the crimes? How close to property?) Totality of the circumstances test just as it sounds. All the above plus nature, condition, location of the land and any other relevant factors. Balancing test* balances the foreseeability of harm against the burden of imposing duty. The calculation basically functions like Learned Hand (balancing the foreseeability of harm, the interest of the business proprietors and their customers, the burden of precaution, the gravity of the harm). *Court chooses this test to determine [lack of] duty. Realm of the Jury? Court says they must balance foreseeability and gravity of the crime to determine if there is a duty (normally these would be used by jury to determine if duty was breached) A.W. v Lancaster County School District (NE 2010) P's son was sexually assaulted in bathroom of Ds school by uninvited man on campus. Did the school owe a duty to protect student from danger of sexual assault and was that danger reasonably foreseeable?

30

P must show a duty owed by D to P, a breach of such duties, causation and damages to recover in negligence action. Whether legal duty exists for actionable negligence is a question of law dependent on facts, but whether evidence establishes such a breach is one of fact. Extent of foreseeable risk depends on facts of case and should be left to trier of fact unless no reasonable minds could differ. Under the 3d ROT, foreseeable risk is an element in the determination of negligence, not legal duty

3d ROT: The status-based duties for land possessors are not in harmony with modern tort law. This Chapter rejects the status-based duty rules and adopts a unitary duty of reasonable care to entrants on the land. At the same time, this Chapter reflects a policy-based modification of the duty of land possessors to entrants on the land whose presence is antithetical to the rights of the land possessor or owner [trespassers]. Side note: the Restatement distinguishes duty of land possessors from the broader category of premises liability. Those who control land have a duty to make it safe control is more important than possession and ownership status Exception: former possessors may be liable if they created or failed to disclose a danger Restatement 49 54. 49. A possessor of land is a person who occupies the land and controls it. 50. A trespasser is a person who enters or remains on land in the possession of another without the possessor's consent or other legal privilege. 51. Subject to 52, a land possessor owes a duty of reasonable care to entrants on the land with regard to: (a) conduct by the land possessor that creates risks to entrants on the land (b) artificial conditions on the land that pose risks to entrants on the land; (c) natural conditions on the land that pose risks to entrants on the land; and (d) other risks to entrants on the land when any of the affirmative duties provided in Chapter 7 is applicable. o 52. Duty of Land Possessors to Flagrant Trespassers (a) The only duty a land possessor owes to flagrant trespassers is the duty not to act in an intentional, willful, or wanton manner to cause physical harm.

31

(b) Notwithstanding Subsection (a), a land possessor has a duty to exercise reasonable care for flagrant trespassers who reasonably appear to be imperiled and (1) helpless; or (2) unable to protect themselves. 53. Duty of Lessors. Except as provided in 52, a lessor owes to the lessee and all other entrants on the leased premises the following duties: (a) A duty of reasonable care under 51 for those portions of the leased premises over which the lessor retains control; (b) A duty of reasonable care under 7 for any risks that are created by the lessor in the condition of the leased premises; (c) A duty to disclose to the lessee any dangerous condition that satisfies all of the following: (1) it poses a risk to entrants on the leased premises; (2) it exists on the leased premises when the lessee takes possession; (3) it is latent and unknown to the lessee; and (4) it is known or should be known to the lessor; (d) A duty of reasonable care for any dangerous condition on the leased premises at the time the lessee takes possession if: (1) the lease is for a purpose that includes admission of the public; and (2) the lessor has reason to believe that the lessee will admit persons onto the leased premises without rectifying the dangerous condition; (e) A duty of reasonable care: (1) for any contractual undertaking; or (2) for any voluntary undertaking, under 4243, with regard to the condition of the leased premises; (f) A duty based on an applicable statute imposing obligations on lessors with regard to the condition of leased premises, unless the court finds that recognition of a tort duty is inconsistent with the statute; (g) A duty of reasonable care to comply with an applicable implied warranty of habitability; and (h) A duty of reasonable care to lessees under 40, Comment m, as well as any other affirmative duties that may apply. See Chapter 7. 54. Duty of Land Possessors to those NOT on Possessors land. (a) For artificial conditions or conduct on land that pose a risk of physical harm to persons or property not on the land, the possessor of the land has a duty of reasonable care.

32

(b) For natural conditions on land that pose a risk of physical harm to persons or property not on the land, the possessor of the land (1) has a duty of reasonable care if the land is commercial; and (2) otherwise has a duty of reasonable care only if the possessor knows of the risk or if the risk is obvious. (c) Unless Subsection (b) applies, a possessor of land adjacent to a public walkway has no duty under this Chapter with regard to a risk posed by the condition of the walkway to pedestrians or others if the land possessor did not create the risk. Jury Instructions NY PJI 2:90 Possessor's Liability for Condition or Use of Premises--Standard of Care Duty: The (owner, possessor) of (land, a building) has a duty to use reasonable care to keep the premises in a reasonably safe condition for the protection of all persons whose presence is reasonably foreseeable. In order to recover, the plaintiff, AB, must prove: (1) that the premises were not reasonably safe; (2) that the defendant, CD, was negligent in not keeping the premises in a reasonably safe condition; and (3) that CD's negligence in allowing the unsafe condition to exist was a substantial factor in causing AB's injury. Negligence is the failure to use reasonable care. Reasonable care means that degree of care that a reasonably prudent (owner, possessor) of (land, a building) would use under the same circumstances. Negligence includes both a foreseeable risk of injury to another and conduct that is unreasonable in proportion to the danger. In deciding whether CD was negligent, you must weigh the likelihood and seriousness of the risk of injury against the burdens involved in maintaining the premises. In other words, what precautions, if any, would a reasonable and prudent person take under the circumstances?

Duty of Government Entities

Holmes on sovereign immunity: sovereign is exempt from suit not because of any formal conception or obsolete theory but on the logical and practical ground that there can be no legal right as against the authority that makes the law on which the right depends Federal Immunities (2d ROT 895A) (1) Except to the extent that the United States consents both to suit and to tort liability, it and its agencies are immune to the liability. (2) Statutory provisions give the requisite consent to suit and liability for many types of tortious conduct. Remedies: (1) Constitutional Guarantees Bivens, (2) Statutory Rights of Action 42 USC 1983 (Civil Rights Action for deprivation of rights); (3) federal torts claims act

33

State Immunities (2d ROT 895B) (1) A State and its governmental agencies are not subject to suit without the consent of the State. (2) Except to the extent that a State declines to give consent to tort liability, it and its governmental agencies are subject to the liability. (3) Even when a State is subject to tort liability, it and its governmental agencies are immune to the liability for acts and omissions constituting: (a) the exercise of a judicial or legislative function, or (b) the exercise of an administrative function involving the determination of fundamental governmental policy. (4) Consent to suit and repudiation of general tort immunity do not establish liability for an act or omission that is otherwise privileged or is not tortious. Remedies: (1) Tort Claims Acts of each state lay out possible claims, procedures; (2) Some rights of action created by federal law are cognizable in state court Privileges (2d ROT 890) One who otherwise would be liable for a tort is not liable if he acts in pursuance of and within the limits of a privilege of his own or of a privilege of another that was properly delegated to him. Examples of privileges that would otherwise be tortious: Police officer acting with reasonable cause has privilege to enter on your land (otherwise, it would be trespass). Health district can quarantine you if you have a contagious and serious disease (otherwise, it would be arrest). Errors in good faith are not actionable, but excessive force may be actionable (i.e. they abused their privilege). Even if you have a privilege, abuse of that privilege may give rise to a claim. Absolute Privileges - No matter how wrongful, you still cannot overcome. Examples: statements made during the course of judicial proceedings by judges, attorneys, witnesses, etc. Public prosecutors are absolutely immune for misusing the office to: initiate, institute, or continue criminal proceedings against a person. Imbler v. Pachtman (U.S. 1976). However, prosecutors can be disciplined as lawyers or criminally prosecuted themselves. As for civil actions against prosecutors, youd have to prove that the misuse occurred before there was probable cause (once there is probable cause, the prosecutor has absolute immunity). Prosecutors also have qualified immunity for preprosecution conduct.

34

No immunity arising from other, non-prosecutorial, nonjudicial functions. e.g. discrimination by judges, prosecutors, and other governmental officials as employers is actionable. In the news: 35 years after Imbler v. Pachtman, Whites reluctant concurrence has come up in Connick v. Thompson. Now the Supreme Court has to decide whether there is this exception to prosecutorial immunity for violations of constitutional rights (e.g. prosecutorial suppression of evidence). White makes it clear that the common law principle of prosecutorial immunity is to protect the judicial process, but allowing for constitutional violations would equally threaten the judicial process. Subordinate Entities If the state is immune/not immune, then that also covers the included municipalities, agencies, etc. Typically no liability for failure to provide general public protective measures for the public at large Governmental v. Proprietary Continuum Proprietary The government is acting like a private actor would be subject to regular liability (e.g. the govt is your landlord) Governmental The spectrum extends gradually out to more complex measures of safety and security for a greater area and populace, whereupon the actions increasingly, and at a certain point only involve governmental functions and would not be subject to regular liability (e.g. defense) Example: Starting from proprietary and ending governmental: Transportation; Housing; Health care; Roadways, bridges & tunnels; Police & fire protection; Public health & environmental protection; Defense Riss v. City of New York (NY 1968) p 230 A municipality is not liable for failure to provide specific protection to a member of the public who was repeatedly threatened with personal harm and eventually suffered dire personal injuries (when her exboyfriend hired someone to throw acid on her face after she had repeatedly requested protection from the police). Court says it is legislative policy decision to determine how to allocate limited resources most efficiently (and judicial incompetence should not interfere with sovereign immunity). Not an operational decision, because reading it as so would allow liability anytime anyone is a victim of crime due to police not providing adequate protection. No special relationship, and did not contribute to harm.

35

Dissent: why defer to the legislature if theyre ineffectual? It would have been simple to inquire into danger, and if it deemed necessary, provide protection. They should be forced to act as a reasonable man under the circumstances. Plaintiffs proposed standard (rejected by the court): State requires: (1) persons who suffer foreseeable injury at the hands of criminals; (2) where police protection has been requested and is unreasonably withheld or withdrawn by the sovereign or its agent; (3) are entitled to reparations for such injury. Schuster v NYC: P provided info about a criminal he saw on an FBI flyer. He was threatened and soon killed. Court found police had a duty to protect him because gov was an active participant in harm, req assistance from civilians. Friedman v. State of New York (NY 1986) p 247 State failed to bring evidence to justify why it took over three or five years (in Muller and Friedman, respectively) for it to take action on its highway safety study. Rule: When the State is made aware of a dangerous highway condition and does not take action to remedy it, the State can be held liable for resulting injuries p. 249. Friedman & Muller The state breached its duty by unreasonably delaying action to remedy a known dangerous highway condition. Cataldo. The state did not breached is duty by coming to the wrong conclusion in its traffic studies. Govt non-delegable duty to plaintiffs: to keep the streets in a reasonably safe condition. This, however, does not mean that the courts can intrude into the govts planning and decision-making functions. To accept a courts verdict as to the reasonableness and safety of a plan of governmental services and prefer it over the judgment of the gov body which originally considered and passed on the matter would be to obstruct the normal governmental operations and to place in inexpert hands what the Legislature has seen fit to entrust to experts. Other ways that a govt can be held liable: when its plan is inadequate, when there is no reasonable basis for its plan, when it fails to take reasonable steps to remedy a problem after it is aware of it, or when it fails to continually review a plan after it has been implemented.

36

[A]n unjustifiable delay in implementing the plan constitutes a breach of the municipalitys duty to the public just as surely as if it had totally failed to study the known condition in the first place (p. 249). Is there really a difference between the discretion it takes to make a decision and the discretion it takes to implement those decisions? The closer the government comes to proprietary functions, then the more susceptible they are to negligence claims. The court can decide to immunize DOT entirely, or they can make decisions to hold them accountable for such unreasonable behavior. Laurer v City of New York: MEs prelim report suggests child dies of blunt force trauma. Later ruled natural causes, but ME fails to correct report so investigation into father continues for years. Discretionary acts (exercise of authority vested in the public official) are not a source of liability. Ministerial acts (acts required in accordance with a governing rule) may, but not necessarily: that a wrong is ministerial merely removes immunity. To recover damages, there must be a duty. And that duty must be to a specific person, not to society at large. There was a statutory violation here, but its intent was to protect society at large, not P. Dissent: To immunize an employee who is the sole possessor of knowledge and power to fix a problem incents people to keep this stuff secret. Zone of proposed duty would be prudently limited still. The Cuffy v New York Special Relationship p. 236 General rule is no tort duty to provide police protection, except in cases of special relationshipthe elements of which are: an assumption by the municipality through the promises or action, of an affirmative duty to act on behalf of the party who was injured; knowledge on the part of the municipalitys agents that inaction could lead to harm; some form of direct contact between the municipalitys agents and the injured party; and that partys justifiable reliance on the municipalitys undertaking If these are met, police can be held liable when they commit to provide protection and fail to do so

Ministerial (no immunity) v. Discretionary (immunity)


The question tree of the govt discretionary function exemption. Was the governments action discretionary or ministerial?

37

Discretionary (See Cope) Was the govt employee acting on the basis of statute or regulation (i.e. was he bound by specific performance)? Yes. He had no choice. o Did he comply with the statute? Yes. No claim. No. Might have a claim (still need to prove negligence, causation, etc.) No. Was his action made on the basis of garden-variety discretion or discretion fraught with policy considerations? o Garden variety. Might have a claim (still need to prove negligence, causation, etc.) o Policy discretion. No claim. Govt immune.

Ministerial (See Wilson, Lauer) Was there a specific duty to the plaintiff (re property or personal interest) as opposed to the agents general duty to the public? No duty. No claim. Yes. Might have a claim (still need to prove, negligence, causation, etc.) Wilson v. City of Jersey City (NY 2010) Calls to Jersey City's 911 one during the bloodshed itselfdid not promptly bring response services due to mistakes in address. Court decided that the dispatchers and the police officers discretionary actions are things that they will not micromanage. As for the call-takers, she says were going to get someone over there as soon as possible. With that, they are undertaking to perform an action, and through that has a duty. How was her conduct distinct from that of the policemen? The policemen were making decisions on how long to search based upon limited resources (discretionary). The call-taker, on the other hand, failed to do basic administrative responsibilities (ministerial). A public entity or employee of such bears the burden to plead and prove immunity under the Tort Claims Act (p. 10 of pdf).

38

Ds defend their actions were discretionary (i.e. competing with other policy goals of their govt agency and subject to resource constraints). But, except for the police and dispatcher, we are unconvinced that they were performing anything besides ministerial acts. As such, if those acts or omissions were negligently done and proximately linked to a plaintiff's harm, it would expose one or more of them, as well as their public employer, to liability and damages, notwithstanding the TCA (p. 9 of pdf). If this case were brought in NY, it would have been dismissed on the pleadings. Allows suit for any negligent act of an officer of the state in the scope of his employment or office where the US would be liable if it were a private actor according to the law of the place where the act occurred The D is the US (stands in the shoes of its agents, offices, employees) Judicial or Legislative acts are immune, as are discretionary functions Logistics: Forum, procedure, evidence rules: all federal No jury trials, no punitive damages, no prejudgment interest, counsel fees limited to 25%. Exemptions: Intentional torts when an employee is acting outside of its scope of duties Does not include constitutional rights violations (you can sue when a govt employee intentionally violates your constitutional rights). o Example, Connick v. Thomson. o Typically, however, its hard to prevail when you try to stretch the 14th Amendment or DP clause to make the govt liable to protect you against private party assailants (e.g. in enforcing restraining orders). Discretionary function: No liability based on an act or omission of an employee of the Government, exercising due care, in the execution of a statute or regulation, whether or not such statute or regulation be valid, or based upon the exercise or performance or the failure to exercise or perform a discretionary function or duty on the part of a federal agency or an employee of the Government, whether or not the discretion involved be abused. 2680(a).

Federal Torts Claims Act

39

New

Cope v. Scott (DC Cir. 1995) p 252 brought under FTCA, plaintiff argued govt: failed to appropriately and adequately maintain the roadway, and failed to place and maintain appropriate and adequate warning signs along the roadway. Discretionary Function Exception applies to decision to place resurfacing of road at 33 of 80 projects in priority, but not to inadequate signage, which once they started erecting had duty to do so adequately Short Rule: The govt must prove that the primary foundation of the decision was policy, not that policy was simply taken into consideration when the decision was made (otherwise, every discretionary act would be immune ant the FTCA would be totally ineffectual). The two-step test to figure out if the claim is exempt based on the discretionary function exemption: o (1) Does a statute or regulation specifically prescribe a course of action for the employee to follow? The employee technically had no choice so if he followed the directive, no claim. If he failed to follow the directive, however, then the govt is open to suit. o (2) If theres no specific performance and the employee does have a choice, then was the decision based on the exercise of political, social, or economic judgment, or public policy judgment? That is, was the employees discretionary act of the very nature and quality that Congress intended to shield from tort liability with the FTCA? Jersey T.C.A. permanent loss of bodily function Injury same as if private person had inflicted No damages against public entity except in permanent loss of function/disfigurement/dismemberment and medical over $1K Brooks v Odom added substantial to permanent loss of bodily function (harder for Plaintiffss) Collins v. Union County Jail (NJ 1997) No damages are allowed under NJ Tort Claims Act except from permanent loss of bodily function, permanent disfigurement or dismemberment Court says psychological harm from rape by prison guard is substantial, permanent loss of bodily function

40

Looking to the intent of the legislature as compared to other, similar statutes (e.g. in workers compensation claims you are allowed to recover for psychological harms)

The Duty Requirement: Non-physical injuries


Common Law Stiff Upper Lip Rule common rationales for no duty rulings on pure emotional harm Emotional disturbance is less objectively verifiable Easy to feign, exaggerate, or to engage in self-deception about the existence or extent of such harm Overwhelming burden: a single act can affect a substantial population School or hard knocks: some minor or modest emotional harm is endemic to living in society and individuals must learn to accept and cope with such harm Malingering and self-indulgence: giving legal credence to and permitting recovery for emotional disturbance may increase its severity Intent, defined (3d ROT 1) A person acts with the intent to produce a consequence if: a) the person acts with the purpose of producing that consequence; or b) the person acts knowing that the consequence is substantially certain to result. Recklessness, defined (3d ROT 2) A person acts with recklessness in engaging in conduct if: a) the person knows of the risk of harm created by the conduct or knows facts that make that risk obvious to anyone in the person's situation, and b) the precaution that would eliminate or reduce that risk involves burdens that are so slight relative to the magnitude of the risk as to render the person's failure to adopt the precaution a demonstration of the person's indifference to the risk. Intentional or Reckless Infliction of Emotional Disturbance (3d ROT 45): An actor who by extreme and outrageous conduct intentionally or recklessly causes severe emotional disturbance to another is subject to liability for that emotional disturbance and, if the emotional disturbance causes bodily harm, also for the bodily harm. Negligent Infliction of Emotional Disturbance (3d ROT 46): An actor whose negligent conduct causes serious emotional disturbance to another is subject to liability to the other if the conduct: a) places the other in immediate danger of bodily harm and the emotional disturbance results from the danger; or b) occurs in the course of specified categories of activities, undertakings, or relationships in which negligent conduct is especially likely to cause serious emotional disturbance.

41

Baseline rule: No recovery for negligently inflicted pure emotional distress Physical Impact Rule You have to receive some kind of physical impact first before you can recovery for emotional harms. Zone of Danger Rule The court scraps the physical impact rule, which had disintegrated, for the zone of danger rule, but limits it to emotional distress that is manifested in bodily injury or sickness (basically moved the physical requirement from the beginning to the end. Advantages to the zone of danger: limits the number of trivial claims, theres foreseeability for defendants Falzone v. Busch (NJ 1965) p 264 wife claims emotional harm from being in close vicinity to area where her husband was hit by a car. P may recover for bodily injury or sickness resulting from fear for her safety caused by a negligent defendant, where the plaintiff was placed in danger by such negligence, although there was no physical impact. Ward v West Jersey and Seashore RR Co: Reasons for denying liability are no longer tenable, decided as a matter of law what is better decided by medical evidence. Problems of tracing casual connection between negligence and injury are not unique to cases without physical impact. Physical Manifestations of Emotional Harm (Physical Impact on the back end) For an emotional distress claim, there has to be evidence of physical impact (beyond simple physical contact). That is, the emotional distress has to be manifested in diagnosable symptoms (e.g. clinical depression, anxiety). What if your distress is real, but its caused by paranoia? e.g. HIV/AIDS Metro-North v. Buckley (US 1997) p 273 pipefitter exposed to asbestos, afraid of getting cancer sues for emotional distress due to increased fear and anxiety P has no physical harm (yet) as no cancer has been diagnosed. As for emotional harm, plaintiff does not fit into either zone of danger rule or physical impact rule, and there is no immediacy/single event that caused his emotional distress. Thus, the only thing he can recover for are monitoring costs for the cancer. Immediacy makes causation clearer, but the problem with cancer is that injuries are incurred over prolonged exposure

42

Where the law is not explicit about the harm, common law principles reign, and in the common law, purely emotional distress (i.e. when plaintiff is disease and symptom free) is not recoverable. What if he never gets the disease? Why cant he recover once now for the emotional distress and later for the disease (if he gets it)? Federal Employers Liability Act (FELA) Not limited to federal employees/employers. Its a federal act, which is why it has that name. But since congress only has jurisdiction to regular interstate commerce, the employee had to have been injured working on something related to interstate commerce (e.g. the interstate railroad v. the NYC subway). It eliminated many common law doctrines (e.g. assumption of risk, you knew this job was dangerous when you took the job). Since then, however, the common law realm of work-related injuries has been largely replaced by an administrative remedy known as workers compensation (which is a no-fault action). Norfolk and Western v. Ayers (US 2003) asbestosis sufferer brings suit for emotional distress due to increased fear of cancer, recovers damages. Asbestosis constitutes sufficient bodily harm to recover on the emotional distress.

Potter v Firestone Tire and Rubber: in the absence of physical injury or illness, damages for fear of cancer can be recovered only if P proves: As a result of Ds negligence, P is exposed to toxic substances which threaten cancer Ps fear stems from knowledge, corroborated by reliable med or sci opinion, that cancer will more likely than not develop from exposure Breach v. Damages Breach (Objective Standard): the kind of conduct that would cause distress in persons of ordinary fortitude (only for negligence, doesnt include intentional torts harming those you know are vulnerable) Damages (Subjective Standard aka The Eggshell Rule): Once you get passed the objective standard, if the actual victim suffered more than the average harm, then you are responsible for the actual harm suffered. Rationale: once a defendant engages in an activity that created a duty under the objective rule, they should be responsible for all harm actually suffered as a result of their action.

B. Witnessing Injury to Others

43

CL is more amenable to allowing negligently inflicted pure emotional distress claims if they are connected with physical injury (even if the injury is suffered by someone else). This is not a derivative claim; the by-stander has a separate claim from the one physically injured (even though some courts have limited recovery based upon the contributory negligence of the victim or the by-standers negligence, if it was somehow involved). This is not punitive damages: you are compensating the bystander for the emotional harm he or she suffered. This is not the same as being in the Zone of Danger. You were not frightened for your own safety (as in Falzone); you are only recovering for the emotional harm you suffered from watching your loved one be physically injured. Contemporaneous Observance of Injury to Close Relative (3d ROT 47): An actor who negligently causes serious bodily injury to a third person is subject to liability for serious emotional disturbance thereby caused to a person who: a) perceives the event contemporaneously, and b) is a close family member of the person suffering the bodily injury. Portee v. Jaffee (NJ 1980) p 286 Mom sees son trapped and killed in elevator General sorrow v. Trauma of watching a loved one die. Must make distinction, otherwise, everyone who had a relative die would be filing a claim. NJ Criteria: death or serious physical injury of another caused by defendants negligence a marital or intimate familial relationship between plaintiff and the injured person observation of the death or injury at the scene of the accident resulting severe emotional distress NY Criteria: Add that you must also be in the Zone of Danger And must be immediate family member Johnson v. Jamaica Hospital (NY 1984) p 295 Parents sue for emotional distress after baby kidnapped from hospital for 4 months. Rights exist correlative with duties. Just because you have suffered doesn't mean that someone else owes you a duty. Must be within zone of danger and D must be shown to have owe a duty of care, and it must be foreseeable that injury would cause P mental distress for a claim to exist Dissent: don't we vindicate wrongs even if there will be a flood of litigation?

44

Gammon v Osteopathic Hospital of Maine: P's father dies in the hospital. Instead of a bag of personal effects, he is given a bag containing a severed leg. If we think there is such a thing as psychological harm at all, we shouldn't arbitrarily require that it be accompanied by something else. This is an artificial device to protect against frauds: the trial process can do that for us, along with the tort principle of foreseeability. Families of the recently deceased are especially vulnerable. Recovery has been allowed for mental distress alone for negligent mishandling of corpses. Psychological injuries do not require evidence of an underlying tort, or physical harm, but are restricted to negligently inflicted severe emotional distress.

C. Economic Loss
Pure Economic loss: pecuniary harm not resulting from physical harm or contact to person or property of plaintiff 3d ROT 1st draft Econ Loss Rule: in general there is no liability in tort for pure economic loss caused unintentionally and without dishonesty or disloyalty 3d ROT 1 2d draft liability for negligent infliction of econ loss: there is no general duty to avoid infliction of economic loss on another Duties of care where: (1) the risk of loss is better regulated by the law of tort than contract between the parties or another body of law; (2) liability would be determinate and proportionate to actors culpability; (3) parties intention or sound party are evident Indeterminate liability may unreasonably burden business, contractual relationships can better allocate risks by agreement, and liability that bankrupts businesses does little to compensate victims of negligence Professional services exception: medical malpractice, legal malpractice, accountants liability Duties of care generally imposed by law not contract, prof standards of care are well developed, and clients/patients are in a weak bargaining position 3d ROT 5 Negligent Performance of Services: (1) one who in the course of his business, profession or employment or in any other transaction in which he has a pecuniary interest, provides a service for the benefit of others, is subject to liability for pecuniary loss caused to them by their justifiable reliance on the service, if he fails to exercise reasonable care in performing it; (2) this liability is limited to loss suffered: (a) by the person or one of a limited groups of persons for whose benefit the D provided services (b) through reliance upon it in a transaction that the D intends to influence

45

Nycal v KPMG Pent Marwick (MA 1998) There's no legal duty for accountants to ensure commercial decisions of nonclients where the accountant did not know a third party would rely on their work in making investment decisions. Balances indeterminate liability of foreseeability test and restrictiveness of near-privity rule Foreseeability: account is liable to any person whom the could reasonably have foreseen would obtain and rely on their opinion, including known and unknown persons (Palsgraf) Near privity test: Cardozo in Ultramares: Limits liability to those whom they are in privity, or approaching privity with R2T 552. Negligent Misrepresentation one who in a traction where they have a pecuniary interest supplies false info for the guidance of others in business transactions is liable for loss caused by their justifiable reliance on the info, if he fails to exercise reasonable care in obtaining or communicating the info Limited to groups of persons whose benefit and guidance he intends to supply the info or knows that the recipient intends to supply it to Through the reliance on it he intends the info to influence Balances indeterminate liability of foreseeability test and restrictiveness of near-privity rule 532 Madison Ave Gourmet Foods v Finlandia Center (NY 2001) Multiple blocks were closed for 2 weeks, with businesses closed as long as 5. P seeks damages for economic loss due to negligence as well as public nuisance. Tort is a means of apportioning risks and allocating burden of loss. Foreseeability alone does not make a duty: you need something running directly to the injured party. Landowners who engage in activities that might injure neighbors are obligated to use reasonable care not to injure them, this doesn't extend to protecting them from loss of business. Oil Pollution Liability Act: 33 USC 2702 (a) in general each responsible party for a vessel or facility from which oil is discharged, into or upon navigable waters or adjoining shorelines, is liable for the removal costs or damages, result from such an incident. 2703: Defenses: (1) Act of God; (2) Act of war; (3) act of a 3rd party: except Where 3rd party is employee, agent, or one in contractual relationship with responsible party and spill is caused solely by 3rd party and responsible party exercised due care to foreseeable consequences 2705: Interim Payments: responsible party shall est a procedure for payment or settlement of claims for interim short term damages BP, Exxon Valdez cases Damages for:

46

loss of subsistence use of natural resources, which shall be recoverable by any claimant who uses natural resources which have been injured, destroyed, or lost, without regard to the management of resources equal to loss of profits or impairment of earning capacity: due to injury destruction or loss of real property, personal property, or natural resources

Ch. 5

Causation

A. Cause in Fact
Factual Cause: (3d ROT 26): Tortious conduct must be a factual cause of harm for liability to be imposed. Conduct is a factual cause of harm when the harm would not have occurred absent the conduct. May also be factual cause under: 27: if multiple acts occur, each of which under 26 alone would have been a factual cause of the physical harm at the same time in the absence of the other act(s), each act is regarded as a factual cause of the harm Burden of Proof (3d ROT PH 28): (a) Subject to Subsection (b), the plaintiff has the burden to prove that the defendant's tortious conduct was a factual cause of the plaintiff's physical harm. (b) When the plaintiff sues all of multiple actors and proves that each engaged in tortious conduct that exposed the plaintiff to a risk of physical harm and that the tortious conduct of one or more of them caused the plaintiff's harm but the plaintiff cannot reasonably be expected to prove which actor caused the harm, the burden of proof, including both production and persuasion, on factual causation is shifted to the defendants. Keep in mind: The plaintiff has the burden of proof of proving causation between the tortious conduct and the harm. Not just the conduct and the harm. Negligent conduct has to be the cause (not just some extraneous situationthat happens to overlap with someones negligencethat caused the harm). Even if your conduct caused the harm, that doesnt mean the plaintiff is entitled to compensation (only if the wrongful conduct caused the harm). Preponderance of the Evidence PJI 1:60 General Instruction Burden of ProofWhen Burden Differs on Different Issues:

47

To say that a party has the burden of proof on a particular issue means that, considering all the evidence in the case, the party's claim on that issue must be established by a fair preponderance of the credible evidence In order for a party to prevail on an issue on which he or she has the burden of proof, the evidence that supports his or her claim on that issue must appeal to you as more nearly representing what happened than the evidence opposed to it. But-For Test: The counter-factual hypothetical question. Youre asking a question about something that never happened. What WOULD have happened had the Ds bad behavior been different enough to be acceptable? Five Steps (from David Robertson) 1) Identify the injuries for which redress is sought 2) Identify the wrongful conduct (what was the omitted precaution) 3) Mentally correct the wrongful conduct to the extent necessary to make it lawful, leaving everything else the same 4) Ask whether the injuries would still have occurred had the defendant been acting correctly in that sense 5) Answer the question. Substantial Factor Test (The Modified But-For Test): Substantial factor standard; distinguishing cause from exacerbation No longer have the strict but for causation. Medical causation was appropriately demonstrated by proof that exposure to the defendants product was a substantial factor in causing or exacerbating the disease. James v. Bessemer, 1998. See also, Zuchowicz notes. The jury has to determine how substantial the defendants negligence is to the injury suffered. The question is no longer what was the cause in the first place, but how much it was exacerbated by the negligent conduct. Substantial factor causation requirement (Connecticut Law of Causation Zuchowicz) Defendants negligent act or omission was a but for cause of the injury The negligence was causally linked to the harm Defendants negligent act or omission was proximate to the resulting injury. N.J. Model Jury Charge 5.34 g

48

By proximate cause is meant that the defect in the product was a substantial factor which singly, or in combination with another cause, brought about the illness. If the product does not add to the risk of the occurrence of the particular illness and hence was not a contributing factor in the happening of the illness then plaintiff has failed to establish that a particular product defect was a proximate cause of the accident. Evidence = what is worthy of belief (the jury decides that) Expert Testimony PJI 1:90 Experts assist in understanding facts, not required to accept testimony, are under same rules re reliability as regular witnesses Supplements lay opinion from competence, direct observation, and common knowledge with education, training and experience Opinion Testimony and the Judge as Gatekeeper for expert testimony, judge focuses on the methods not the conclusions, shaky but admissible is okay A trial judge is required to (1) assess the evidence for reliability and admissibility and (2) assess the evidence for sufficiencythat its enough for a reasonable jury to find upon. Creanga v. Jardal and Lucent Technologies (NJ 2005) Prejudicial v probative value, reliability, relevance, fact witness: direct observation or expert, competence of witness Differential diagnosis: rule in all plausible causes for condition, which are generally capable of causing the injury, rule out those that did not cause it, and conclude the most likely cause o Employ diagnostic sufficient techniques to have good grounds for conclusion Under Daubert v Merrell Dow Pharm, trial judges are charged with ensuring that expert testimony both rests on a reliable foundation and is relevant to the task at hand. Supreme Courts standard in identifying admissibility of expert testimony: Whether the theory can be (and has been) tested according to the scientific method been subjected to peer review and publication (i.e. does it have support from the field?) In the case of a particular scientific technique, the known or potential rate of error Whether the theory is generally accepted Abuse of dicretionary standard on appeal (Joyner)

49

rigor customary in the field. Kuhmo v Carmichael (1997) objective of Daubert is to ensure reliability and relevance of expert testimony. Rule 702: Testimony by Experts: Pre-Daubert: if sci, technical, or other specialized knowledge will assist trier of fact to understand evidence or determine a fact in issue, a witness qualified as expert may testify in form of opinion or otherwise Post-Daubert: if (1) testimony is based on sufficient facts/data; (2) is the product of reliable principles and methods; and (3) the witness has applied the principles and methods reliably to the facts of the case Proving facts through professional opinions: No verbal formula, no magic words, but courts tend to use the preponderance while scientific community uses probability. General Acceptance test, of Landrigan v Celotex, selfvalidating a technique or method by group acceptance, was rejected The experts opinion must reflect an acceptable level of professional certainty, but notions of reasonable degree of medical certainty are outdated and are not insightful (yet courts still continue to use them) Strict standards for evidence if you dont have them, then you get parade of horribles (i.e. frivolous lawsuits) Counter-arguments: the idea of scientific certainty is a myth Standards of Review on Appeal Questions of law De novo review (the appeals court can ignore the trial courts holding) Questions of fact Defer to the jury if there is more than a scintilla of evidence Unless no reasonable person could come to the conclusion that the fact-finder reached, then you leave it alone. Review facts in the light most favorable to the party that prevailed at the level below. You dont ignore rationality but keep in mind that your opinion isnt the only reasonable one. Questions of admissibility of evidence Abuse of discretion standard The judge is making fact-sensitive determination (so unless he abuses the discretion, you let it stand)

50

Does a piece of evidence, if accepted, lead a factfinder to find for one party over the other? Is it prejudicial? Is it an appeal to bias or passion? General Causation v Specific Causation (disease) It is able to cause the effect v. it did in fact cause the effect here Relative Risk Ratio (SMR) Compare the rate of occurrence of the injury/disease in the general population or control group with the rate of occurrence in the population exposed to the risk factor or causal agent. 2.0 ratio as threshold (doubling risk) establishment of single quantitative factor to determine causation usurps jurys role, which is to consider ALL relevant factors to determine whether burden of causation has been met Landrigan v Celotex: relative risk of 2.0 isnt a password to finding causation, but is 1 piece of evidence. Court functions to distinguish scientifically sound reasoning from that of the selfvalidating expert, presenting unsubstantiated personal beliefs Biological Model (Epidemiological Evidence) subjective judgments in determining causation of medical issues Surgeon General (1964): Causation = a significant, effectual relationship between an agent and an associated disease or disorder in the host. Distinct from correlation or coincidence. Do not have to disprove all other possibilities P proves that it was a substantial factor in the injury and then burden shifts to D to disprove (or provide supervening causes). Stubbs v. City of Rochester (NY 1919 Cardozo) p 340 Man drank contaminated water, gets typhoid. Circumstantial evidence to prove causation. Liability allowed where it can be said with reasonable certainty that Ds conduct was the direct cause Burden of proof of cause of fact General cause was clear, but plaintiff had to prove specific cause using evidence of work place exposure, habits, etc. Zuchowicz v. United States (2d Cir. 1998) p 347 Danocrine overdose leads to heart attack Must prove that the negligence (i.e. the overdose) and not just the drug itself caused the death. Thanks to Judges Cardozo and Traynor, you can establish that a negligent act caused the harm, if (1) the act increased the chances that a particular type of accident would occur and (2) a mishap of that very sort did happen.

51

Where such a strong causal link exists, it is up to the negligent party to bring in evidence denying but for cause and suggesting that in the actual case the wrongful conduct had not been a substantial factor (p. 353). Burden shifting here is not res ipsa but what it has in common with res ipsa is a lack of direct evidence. McDarby v. Merck (NJ 2008) inadequate label warning as to cardiovascular effects of drug Modified but-for test: P must show that Ds action was a substantial contributing factor in causing or exacerbating the injury Hard sell directly to patients. Drug was only proven to be better than placebo. General Rule: Liability flows from knowledge and capability. New discoveries & failure to warn Even if the FDA approves a label, you are still responsible for warning patients and doctors as soon as an association [doesnt have to be causation] between the drug and an unintended harm is discovered. Presumptions If a fact is presumed (e.g. res ipsa loquitur, statutory presumption), it can generally be rebutted. Presumption that if the FDA approved a drug label, then the warnings on the label are adequate. The bursting bubble once you introduce competent evidence which, if accepted, would rebut the presumed fact, then the bubble bursts and the party proposing the fact now has to introduce contrary evidence in order to counter the bubble Whether presumptions have been rebutted are jury questions The Heeding Presumption Its the standard but-for problem. How do you know what would have happened if there was an adequate warning? This is even harder when the victim dies and cant tell you what he would have done. The plaintiffs side presumes that the victim would not have taken the drug if he had adequate warning. Then the defendants have to rebut that presumption. only evidence of habit can rebut the heeding presumption

52

evidence of past negligent acts is inadmissible to prove that you were negligent in another situation (but you may be able to use it to prove knowledge). Bradford Hill Factors: (1) Strength of association how often do the suspected cause and the effect appear together? (2) Consistency has it been repeatedly observed by multiple parties in different places? (3) Specificity of effect to certain group how close is the relationship? Is disease limited to specific workers and particular sites/types of disease? (4) Temporality The cause must precede the effect. (5) The Biological Gradient the Dose-Response Curve (the dose makes the poison). When you increase the dose, the symptoms should increase and vice versa. How much exposure is required? (6) Biological Plausibility this is a feature which I am convinced we cannot demand. What is biologically plausible depends on the biological knowledge of the day. Sometimes you just dont know. (7) Coherence how it fits in with other facts (8) Experiment Here the strongest support for the causation hypothesis may be revealed. If you cant do an experiment, then you might have to go backward to see what data is already available. (9) Analogy Once all the leg work is done on an earlier test, you can piggy-back on it with similar drugs/scenarios. We often require more evidence in tort claims to get damages than regulatory agencies would require to take preventative measures.

B. Multiple Defendants
NY PJI 2:70 Proximate cause - if action was substantial factor in bringing about injury that reasonable people would regard as cause of the injury NY PJI 2:71 with multiple causes each is regarded as a cause provided it was a substantial factor in bringing about the injury o Did the Defendant act negligently? o Did the Defendants negligent act cause harm? o What harm (specifically) did the negligent act cause? 2d ROT 433A (1965) on apportioning fault Can only apportion where (1) harms are distinct or (2) there is reasonable basis for determining each causes contribution

53

FELA cases (Fed. Juris., railroad workers) Statute allows full recovery from a defendant as long as the D was negligent and contributed, however slightly, to the injuries (doesnt matter if other actors were more responsible, as long as D was at fault at all he can recover fully (see Ayers v. Norfolk Southern, US 2005) NJ see Dafler v. Raymark (NJ 1993) 45 year smoker with cancer sues for 6 years of exposure to asbestos o Jury finds D is 30% contribution o Supreme Court rules contribution is more question of feasibility of apportioning losses, where a factual basis can be found for some rough practical apportionment, it can be made o If no such basis can be found, defendant is fully liable Joint and Several Liability o Joint Liability Liability shared by two or more parties. Both fully liable. o Several Liability Liability that is separate and distinct from another's liability so that the plaintiff may bring a separate action against one defendant without joining the other liable parties. This avoids having to make parties joint (which would bring up problems of jurisdictions, venues, etc.). If there is concurring negligence among parties, you can collect in full from all parties because they are each fully liable. (if one party pays more than his share, then he can seek recovery from the party who has underpaid). This rule ran into little resistance before the modern era (the last 50 years) because until the modern era, contributory negligence was a full defense, and the plaintiff was not entitled to recover at all. Now, theres comparative negligence and courts have changed the meaning of several liability. It used to mean that several liability meant that you were fully liable, but nowadays, it means that you are partially liable. Legislative tort reform: because insurance companies were having to shell out full liability for only being 1% at fault. See slide 11 for an example of reform in New York statutes. NY CPLR 1601 if a party is less than 50% liable for the total, liability for non-economic losses shall not exceed the partys share of the liability You can still be fully liable for economic damages, but only proportionally liable for the non-economic damages.

54

Different compromises have been struck in different States; you dont need to know the rules in detail, you only need to know that joint tortfeasors are responsible for the entire loss except as modified by comparative negligence and/or tort reform statutes. Joint Liability and Several Liability historically, is a phrase that means severally liable. Liability that may be apportioned either among two or more parties or to only one or a few select members of the group, at the adversary's discretion. Thus, each liable party is individually responsible for the entire obligation, but a paying party may have a right of contribution and indemnity from nonpaying parties. o Joint Liability or Several Liability one or the other (not both)Proportionate several liability one solution to the several liability vocabulary problem. This is making it clear that each codefendant is not fully liable, but rather has a proportion of the liability. Summers v. Tice (CA 1948) p 375 quail hunting accident o Policy notes: the court ultimately decided that it was better for an injured plaintiff to get relief from both parties than the plaintiff to be without remedy altogether. o Burden shifting to the defendants (similar to Ybarra) happens when the plaintiff has no realistic way of figuring out the information (not simply that he doesnt know). The Plaintiff has sufficiently proven causation (as opposed to conclusively proving): the negligence of both persons was the cause of the harm. All negligent actors are before the court. o Breach: They fired a highly dangerous instrumentality without knowing for certain what they were shooting at. o Apportionment The Restatement favors apportionment on the theory that parties should not pay for harms they have not caused. Nevertheless, apportionment can be pretty arbitrary, especially when the proportion of causation doesnt correspond to the proportion of harm. Hymowitz v Eli Lilly Co (NY 1989) p 379 DES cases, pregnancy pill causes cancer later in life and inter-generationally o Look to legislative intent to justify expansion of tort liability for cases with no provable specific causation o Market share theory of liability based on amount of risk presented to public at large, not to individual plaintiff o Tend to focus on creation of risk rather than specific causation in determining liability o Disallowed proof non-causation to exculpate from specific cases o Result is not fundamentally unfair, but is not traditional tort law, which requires causation for the specific injury caused

55

C. Proximate Cause & Scope of Liability (Legal Cause)


In most cases, there are several causes (negligence-related, act of nature related, contributory negligence related, coincidence, a third partys intentional to inflict harm, etc.). o Forget about which one of these is the proximate cause. Rather, was Party As negligence was a substantial factor in causing the loss suffered. NJ Model Civil Jury Charge (1992) a cause which necessarily set the other causes in motion and was a substantial factor in bringing about the accident, a cause which naturally and probably led to and might have been expected to produce the accident complained of NJ revises in 1998 - Whether the harm is so connected with the negligent actions or inactions that you decide it is reasonable that the defendant should be held wholly or partially responsible (seems easier on defendant, allows intrusion of morality) NYPJI 2:70 substantial factor and had such an effect in producing the injury that reasonable people would regard it as a cause of the injury (can be substantial despite relatively small percentage) 3d ROT 29 Liability limited to physical harms that result from risks that made the actors conduct tortious 3d ROT 30 Actor not liable for harm when the tortious aspect of the actors conduct was of a type that does not generally increase the risk of that harm Proximate cause as a limiting factor o Its a question of fact for the jury, but judges use it all the time to dismiss claims on the hearings. o Negligence itself is a limiting principle (limiting claims that would be acceptable under strict liability). o Once you establish negligence, then are you supposed to be responsible for everything that was a result of my negligence? No. Thats where proximate cause comes in as the next limiting factor. o Proximate cause = how we connect the wrong to the law (it has to be sufficiently and logically connected)

D. Unexpected Harm
Eggshell Plaintiff Rule applies to Causation too (not just damages) o But its like the converse of foreseeability rule in duty determinations even though everyones idiosyncrasies are not foreseeable, youre still liable

56

Policy on Scope of Liability What has an element of chance (e.g. hitting someone with a pre-existing condition that gets treated negligently at the hospital and being responsible for damages that result from all three) does not automatically make it unfair. Dont mistake random with unfair. 2d ROT 461 the negligent actor is subject to liability for harm to another even though a physical condition of the other makes the injury greater than that which that actor as a reasonable man should have foreseen as a probable result of his conduct o Benn v. Thomas (Iowa 1994) p 401 plaintiff susceptible to heart attacks, got hit by car, suffered heart attack a few weeks later o You are responsible for the full extent of damages you cause, as long as it is of the kind foreseen that makes your conduct tortious o Is exception to general rule allows recovery when tortious conduct aggravates a previously existing condition o Jury charge as P wanted: If Benn had a prior heart condition making him more susceptible to a heart attack than a person of normal health, then the defendant is responsible for all the injuries and damages which are experienced by Benn, proximately caused by the defendants actions, even though the injuries claimed produced a greater injury than those which might have been experienced by a normal person under the same circumstances. Direct Consequences Test o In re Polemis (KB 1921) p 406 Board drops in hold, causes unforeseeable spark that causes fire Court finds directness of negligence to the damages is sufficient Foreseeability of accident, not foreseeability of result, is relevant to breach Limitation on liability is created by directness requirement Reasonably Foreseeable Test (Reversed Direct Consequences Test) o Foreseeability is relevant to breach: to determine whether an act is negligent, it is relevant to determine whether any reasonable person would foresee that the act would cause damage; if he would not, the act is not negligent. (see Conks version of the restatement, if its not the type of harm to be expected, then the action is not wrongful). o RULE: Even though injury may result from a negligent act, liability for that injury is limited to the risk reasonably to be foreseen. Nature of actual damage/results must be foreseeable o Wagon Mound (Privy Council 1961) p 409 Oil spill causes fire on water. Rejects Polemis, institutes foreseeability rule, finds even directness test is vague.

57

Liability for negligence is no doubt based upon a general public sentiment of moral wrongdoing for which the offender must pay Tort law is to encourage people to take reasonable precautions, foreseeability aids this end but directness does not

E. Unexpected Victim
Palsgraf v. L.I.R.R. (NY 1928) p 425 fireworks packaged, boarding train, woman injured on end of platform o In contemporary terms, Cardozo took the scope of liability issue and decided it as a matter of law. (scope of liability is a Third Restatement term and, of course, the Restatement thinks that scope of liability should be a jury determinationunless, of course, reasonable minds could not differ on it, which is what Cardozo thinks). o Theres no doubt about the but-for argument, so we have to look to proximate cause as a limiting factor. P loses because the risk [to her] was not within the range of apprehension. aka foreseeable Cardozo There must be a duty to the plaintiff Foreseeability the risk must be within the range of apprehension. The risk reasonably to be perceived defines the duty to be obeyed. The judge makes the decision early on as a matter of law (intrudes quite aggressively) Focus on analyzing the wrongful conduct Hes making a policy decision by making a duty determination This approach is rather standardless This approach is also rather standard-less Andrews Theres a general duty to society You should be liable for all the harm that your negligence causes

The jury makes the proximate cause decision, as a matter of fact

Revival of directness over foreseeability (Return to Polemis)

58

Kinsman Cases (2d Cir. 1968) One boat negligently gets detached from its moorings, hits another boat downstream (that was properly moored), and in combination, hit a bridge downstream. The bridge was negligently down (when it should have been up). The accident creates a dam that floods properties upstream. o Should victims of economic suffering from having to find new transportation across the river (since the bridge was destroyed) be able to recover? Though this was a long chain, this was a direct consequence and foreseeable. Ultimately, had to cut off liability at some point though (even though the results were foreseeable). The Restatement on Liability o Restatement (Second) 281 The actor is liable for an invasion of an interest of another, if: (a) the interest invaded is protected against unintentional invasion, and (b) the conduct of the actor is negligent with respect to the other, or a class of persons within which he is included, and (c) the actor's conduct is a legal cause of the invasion, and (d) the other has not so conducted himself as to disable himself from bringing an action for such invasion. Restatement (Third) 29 An actor's liability is limited to those physical harms that result from the risks that made the actor's conduct tortious. 30 An actor is not liable for physical harm when the tortious aspect of the actor's conduct was of a type that does not generally increase the risk of that harm.

Tries to maintain the realm of the jury and so defines duty broadly. It only lets judges determine duty narrowly if policy reasons require it. Given that a case goes to a jury, does the language in the 3d ROT adequately encourage the jury to limit the amount of liability to the type of conduct that increases the risk of that harm? Conks rewrite of the Restatement: Liability within the scope of the risk: An actor is liable only for the general sort of harms the foreseeable risk of which made the actors conduct wrongful. (Conk takes on the NY stance: when you make a duty determination, you dont take foreseeability into account.) Remember, the Restatement believes that the scope of liability is a fact question to be answered by the jury.

59

Their perspective on duty: if your conduct creates a risk, you owe a duty (+ special relationships). The judge should only minimally mess with this standard and declare that there is no duty. If they do, those determinations should be categorical determinations (not just for the specific case, but for all similar cases. That is, they should not delve into the case-specific details.) Restatement says that the foreseeability of the actual injury suffered (in a very specific sense) does not matter. Once youve established duty, breach (negligence), causation, injury, then the you are liable for the actual injury suffered (e.g. The Eggshell Rule, neighbors to the burning house). Policy Restatements objective is to put it in the jurys hands, but creating a minimalist requirement for duty. o Where should the limitations be administered? On Duty or on Scope of Liability? In the jurys hands or in the judges hands? (For the record, concerns about juries are overblown. Theyre just as conservative, and sometimes more so, than judges).

60

F. Superseding Causes
Act of 3d party or other force which precludes defendant from liability. Usually a complete defense, not just a concurrent cause. o Plaintiffs conduct in addition to third parties, the plaintiffs conduct can be considered a superseding cause. In the modern law of comparative fault, if you use plaintiffs conduct as a superseding cause then it looks like you could bar her from all recovery when she would be entitled to at least some recovery based on comparative negligence. 2d ROT 440 A superseding cause is an act of a third person or other force which by its intervention prevents the actor from being liable for harm to another which his antecedent negligence is a substantial factor in bringing about. 2d ROT 441 Intervening Force Defined: An intervening force is one which actively operates in producing harm to another after the actor's negligent act or omission has been committed. 2d ROT 442 The following considerations are of importance in determining whether an intervening force is a superseding cause of harm to another: o (a) the fact that its intervention brings about harm different in kind from that which would otherwise have resulted from the actor's negligence; o (b) the fact that its operation or the consequences thereof appear after the event to be extraordinary rather than normal in view of the circumstances existing at the time of its operation; o (c) the fact that the intervening force is operating independently of any situation created by the actor's negligence, or, on the other hand, is or is not a normal result of such a situation; o (d) the fact that the operation of the intervening force is due to a third person's act or to his failure to act; o (e) the fact that the intervening force is due to an act of a third person which is wrongful toward the other and as such subjects the third person to liability to him; o (f) the degree of culpability of a wrongful act of a third person which sets the intervening force in motion. 2d ROT 442A Where the negligent conduct of the actor creates or increases the foreseeable risk of harm through the intervention of another force, and is a substantial factor in causing the harm, such intervention is not a superseding cause.

61

2d ROT 442B Where the negligent conduct of the actor creates or increases the risk of a particular harm and is a substantial factor in causing that harm, the fact that the harm is brought about through the intervention of another force does not relieve the actor of liability, except where the harm is intentionally caused by a third person and is not within the scope of the risk created by the actor's conduct. o 3d ROT 35. Enhanced Harm Due To Efforts To Render Medical Or Other Aid o Myiamoto v. Lum (HI 2004) If you suffer harm from several parties negligence down a chain, you can hold the first party liable for all the harm suffered since you would have never been subject to the subsequent negligence had it not been for the first one (e.g. you get hit by a car and are taken to the hospital where the doctor negligently gives you incorrect treatment). o Chain of injuries, what is NOT a superseding cause. The doctors negligence is the type of injury that Ds conduct put you at an increased risk for. P wouldnt have had treatment in the first place but for Ds actions. o P would also have a medical malpractice case 2d ROT 448 The act of a third person in committing an intentional tort or crime is a superseding cause of harm to another resulting therefrom, although the actor's negligent conduct created a situation which afforded an opportunity to the third person to commit such a tort or crime, unless the actor at the time of his negligent conduct realized or should have realized the likelihood that such a situation might be created, and that a third person might avail himself of the opportunity to commit such a tort or crime. o Scope of risk the Restatement says that even intervening actors who commit intentional torts are not superseding causes if the harm that occurs is within the scope of the risk. o Foreseeability if you describe the plaintiffs facts specifically, it is unforeseeable, if you describe it generally, it is foreseeable. o Doe v. Manheimer (CT 1989) p 416 Unkempt hedge creates location for rape, assault, victim sues property owner. This harm was not of the same nature as the foreseeable risk created by the property owners omission (even though the tall hedges were the cause in fact for plaintiffs rape) The theory of catalyst liability suggested by plaintiff is far too ambitious. It essentially makes proximate cause and cause in fact one and the same. Court ruled as a matter of law even though there was a question of fact regarding whether the neighborhood was crime-ridden and a criminal might avail himself of the opportunity created by the property-owners bushes

62

2d ROT 449 If the likelihood that a third person may act in a particular manner is the hazard or one of the hazards which makes the actor negligent, such an act whether innocent, negligent, intentionally tortious, or criminal does not prevent the actor from being liable for harm caused thereby. Hinojo v. NJ Manufacturers Insurance Co. (NY 2002) Employee injured on job. Claim against NJM for defect in product safety design, but employer refused to update/modify equipment per manufacturers instruction o Once you have a superseding cause, you dont need to know whether the defendant was negligent or not. o A superseding cause relieves the actor from liability, irrespective of whether his antecedent negligence was or was not a substantial factor in bringing about the harm. Therefore, if in looking back from the harm and tracing the sequence of events by which it was produced, it is found that a superseding cause has operated, there is no need of determining whether the actor's antecedent conduct was or was not a substantial factor in bringing about the harm. Scope Of Liability For Intentional And Reckless Tortfeasors 3d ROT 33(c) an actor who intentionally or recklessly causes physical harm is not subject to liability for harm the risk of which was not increased by the actor's intentional or reckless conduct.

[ADD IN MISSING SECTIONS]

A. Nuisance
Non-trespassory interference with anothers right to use land o Unreasonable interference with anothers use or enjoyment of land o Voluntary conduct that substantially interferes, and it does not require proof of fault Applicable to All Nuisance o Significant Harm There is liability for a nuisance only to those to whom it causes significant harm, of a kind that would be suffered by a normal person in the community or by property in normal condition and used for a normal purpose. o Type of Conduct Essential to Liability The conduct necessary to make the actor liable for either a public or a private nuisance may consist of

63

an act; or a failure to act under circumstances in which the actor is under a duty to take positive action to prevent or abate the interference with the public interest or the invasion of the private interest. Public v. Private Nuisances Private: solely matter of tort Public liability Must interfere with the enjoyment of Must interfere with a right common to land the general public Only land-owners, easement-owners, Any member of the public whose right etc. can bring suit was interfered with can bring suit, but only if the person bringing suit can show special injury as a result of the nuisance (injury or damage of a kind different from that suffered by the rest of the public). The special injury requirement has come under sharp criticism and has been restricted by statute, judicial decisions, and the Restatement.

Public Nuisance o Elements: (1) an unreasonable interference; (2) with a right common to the general public; (3) by a person or people with control over the instrumentality alleged to have created the nuisance when the damage occurred
o Defined 821B an unreasonable interference w/ a right common to the general public. Circumstances that may sustain a holding that an interference with a public right is unreasonable include the following: Whether the conduct involves a significant interference with the public health, the public safety, the public peace, the public comfort or the public convenience, or whether the conduct is proscribed by a statute, ordinance or administrative regulation, or whether the conduct is of a continuing nature or has produced a permanent or long-lasting effect, and, as the actor knows or has reason to know, has a significant effect upon the public right. Remedies Who can recover Damages In order to recover damages in an individual action for a public nuisance, one must have suffered harm of a kind different from that suffered by other members of the public exercising the right common to the general public that was the subject of interference.

64

Injunctions In order to maintain a proceeding to enjoin to abate a public nuisance, one must have the right to recover damages, or have authority as a public official or public agency to represent the state or a political subdivision in the matter, or have standing to sue as a representative of the general public, as a citizen in a citizen's action or as a member of a class in a class action. Elements of Liability the same as private nuisance Rhode Island v Lead Industries (2008) p. 698 Lead poisoning is a public health crisis of particular danger to children Compensatory damages; punitive damages; order requiring D: (1) Abate lead pigment in all RI buildings accessible to children; (2) and fund educational and lead-poisoning prevention programs Private Nuisance (Restatement Black Letter Law) 821 o Related to Land A private nuisance is a nontrespassory invasion of another's interest in the private use and enjoyment of land. o Who Can Sue For a private nuisance there is liability only to those who have property rights and privileges in respect to the use and enjoyment of the land affected, including possessors of the land, owners of easements and profits in the land, and owners of nonpossessory estates in the land that are detrimentally affected by interferences with its use and enjoyment. o Elements of Liability One is subject to liability for a private nuisance if, but only if, his conduct is a legal cause of an invasion of another's interest in the private use and enjoyment of land, and the invasion is either intentional and unreasonable, or unintentional and negligent, reckless, or abnormally dangerous. Intentional acts for the purpose of causing it, or knows that it is resulting or is substantially certain to result from his conduct. Unreasonable the gravity of the harm outweighs the utility of the actor's conduct, or the harm caused by the conduct is serious and the financial burden of compensating for this and similar harm to others would not make the continuation of the conduct not feasible. gravity factors

65

The extent of the harm involved; the character of the harm involved; the social value that the law attaches to the type of use or enjoyment invaded; the suitability of the particular use or enjoyment invaded to the character of the locality; and the burden on the person harmed of avoiding the harm. utility factors the social value that the law attaches to the primary purpose of the conduct; the suitability of the conduct to the character of the locality; and the impracticability of preventing or avoiding the invasion. gravity v. utility conduct is unreasonable if it is malicious or indecent severe and greater than the other should be allowed to bear without compensation avoidable without undue hardship unsuited to the locality (whereas the plaintiffs enjoyment is suited to the locality) Boomer v. Atlantic Cement Co. D operates large cement plant (initial investment $45 mil, over 300 emp). Alleged nuisance to neighbors: dirt, smoke, vibration from plant. [Injuries are actually pretty serious considering they were using explosives to blast in the quarry.] Plaintiffs sought damages + injunction but only got damages. Appellate court granted a limited injunction on top of damages but just lasting long enough to calculate the present and future damages. For all intents and purposes this is not an injunction. o Nuisances should be eliminated when it is feasible o Cost of harm which cant be feasibly avoided should be borne by those who inflict it to pursue their own ends, and generally profit from doing so Dissent Agreed that the injunction should be granted, but didnt think the court should have overruled its rule about granting injunctions. By permitting the injunction to become inoperative upon the payment of permanent damages, the majority is, in effect, licensing a continuing wrong. [whats wrong with that, isnt that what Coase wants?]. Also, once those damages are paid, then the cement company has no incentives to alleviate the wrong in the future (e.g. the technological advances early discussed).

B. Trespass
Trespass v. Nuisance: both torts are strict o Trespass an actionable invasion of a possessors interest in the exclusive possession of his land. Remedy: ejectment.

66

Nuisance an actionable invasion of a possessors interest in the use and enjoyment of his land. Remedy: injunction or damages.

Trespass v. Private Nuisance o Trespass Requires Physical Invasion as opposed to just invasion with use (clearly this distinction is arbitrary) o Unintentional Liability for unintentional trespass = liability for unintentional nuisance o Intentional Liability for intentional trespass is treated like any other intentional tort whereas intentional nuisance considers reasonableness and amount of harm For this reason, intentional trespass claims are much easier to win than intentional nuisance so try to being with evidence that theres been a physical invasion. Trespass to land wrongful entry upon the lands of another, with damage (however inconsiderable) to the real property o The entry is the offense (i.e. the breaking of the close) so the minuteness of the injury doesnt matter (in fact, there can be no harm at all). o Theres no knowledge required; even if dont know youre trespassing, youre trespassing (mistake is not a defense) o Remaining on the land, or failing to remove something is also trespass Trespass is an intentional tort, but there is not necessarily malice or even carelessness o Intentional that means voluntary movement that directs you to the land (not intent in terms of knowing that the land is not yours, even under mistaken belief) Trespass is a fact question as soon as it is determined that you are interfering with a property-owners exclusive control, there is trespass. Remedies o Damages Permanent Damage Judgments work like servitudes When an activity continues with permanent damages, the defendant is essentially buying the right to pollute. And that servitude runs with the land (e.g. when you sell your property, the new owner cant sue again). E.g. Boomer. o Injunction Rule If the plaintiffs injury is slight and that of the public great, then Plaintiff should get damages instead of an injunction so that an ever-advancing civilizations needs not be impeded by one person. Balancing of Equities Comparative Injury o Injury to defendant and the public resulting from the injunction o Injury to plaintiff if the injunction is denied Public omitted from second consideration

67

Isnt the public also generally harmed by the defendants activity if, for example, its something like air pollution? Why not take that into consideration? o Since class actions are often impracticable, it seems appropriate that if a judge will consider harm to third parties by granting an injunction then it should also consider the harm to third parties by not granting the injunction. Proportional But this is not simply a matter of dollars and cents. Even if the defendant will lose more money than has the plaintiff, its a matter of proportional harm to each party. Buying & Selling Injunctions If its still worth more to the defendant to try and dissolve the plaintiffs judgment, he may try and buy/settle for a dollar figure after the fact. This can make injunctive relief more efficient because the parties can bargain over what its actually worth to them (as opposed to the jury/court setting a value for damages). 2d ROT 158 Subject to liability regardless of harm if you intentionally enter the land or cause another person or object to enter anothers land OR remain on land OR fail to remove something you have duty to remove o 164 Liability incurred for intentionally entering land, regardless of mistake in fact or law or however reasonable, unless induced by possessor of land Mistaken belief he possesses the property, he has consent of possessor, or of third party with power to consent, or has some privilege o 165 Unintentional trespass requires actual damage. Intentional trespass is strict liability crime. o 196 Public Necessity can enter if actor reasonably believes it necessary to avert imminent public disaster o 197 - Private necessity privilege to enter land to prevent serious harm to actor, or his land or chattels, or a third persons unless he knows or has reason to know that the one for whose benefit he would enter is unwilling. Not liable for trespass, but liable for any damage done, except where the threat to avert is brought by tortious conduct or contributory negligence of possessor o 211 if legislative enactment carries privilege to enter, then no liability Martin v. Reynolds Metals Co. (OR 1959) p 670 Damage to farm from nearby aluminum reduction plant. Plaintiff could no longer raise livestock because they were all poisoned from the airborne particles from the aluminum plant. o

68

Rule: Size doesnt matter. Intrusion of small particles of aluminum fluoride constituted trespass (as opposed to nuisance). We must look at the character of the instrumentality which is used in making an intrusion upon anothers land we prefer to emphasize the objects energy or force rather than its size. Hard cases make bad law (i.e. results-oriented law is bad law). One reason the court went this way is because the statute of limitations would have barred plaintiff from recovery if they called it a nuisance. However, this is really a nuisance claim because plaintiffs right to use has been interfered with, not right of possession. Strict Liability NOT NEGLIGENCE (D had been using state of the art manufacturing and filtering process, but that did not matter). Since trespass, immaterial if action was careless, wanton, willful or entirely free from fault

C. Police Power
Takings of possessory interests require just compensation. Takings of use interests by police power do not require just compensation. government may act to promote public health, safety or the general welfare a state may impair private property rights to that end IF protection of the public interest predominates over that impairment, the action is valid. No compensation need be paid.

Regulatory taking Lucas v. S. Carolina Coastal Commn (Supreme Court of the United States, 1992) Holding: If the govt is interfering with use of land through regulation/zoning (in this case, banning building on it) then youre entitled to compensation. If its just exercise of the police power, then no compensation. How do you know when something is just an exercise of police power? Its an exercise of police power if at common law the activity that the govt forbids would be considered a nuisance.

69

A. Compensatory Damages
The amount necessary to make plaintiff whole or indifferent to the loss Single payment judgments are the norm mostly due to administrative difficulties. Compensatory damages include o Economic/pecuniary (e.g. medical costs, lost wages, property damage, new job training) and o Non-economic/non-pecuniary (e.g. pain and suffering, disability, impairment, aggravation of pre-existing condition). o special damages property damage (actual cash value), lost/income profits (past and future), medical bills, other repair costs (including job training). o spousal damages gratuitous care rendered to the spouse (market value, e.g. of a nurses care that your wife instead performs but not necessarily her lost wages for leaving her wellpaying job), the impact on the marital relationship, loss of companionship, Punitive damages reserved for malicious, intentional, etc. acts beyond normal breach of duty Incommensurability disappointment in that theres no compensation that will make you completely indifferent to the loss Not relevant to compensatory damages (public policy considerations are not for the jury) Punishment, Deterrence, Redistributing wealth, Helping the less fortunate or the level of the defendants wealth The tort reform movement focuses mostly on reducing damages (the Bronx Jury Myth) NY Verbal Threshold: Permanent consequential limitation of use of a body organ or member; significant limitation of a body function or system; and medically determined injury/impairment of non-perm nature which prevents injured person from performing substantially all of material acts which constitute usual and customary daily activities Toure (NY Ct. App. 2002) In order to determine the extent or degree of physical limitation, an experts determination of a numeric % of Ps loss of range of motion can be used to substantiate a claim experts qualitative assessment may also suffice, provided it has objective basis compares Ps limitations to normal function, purpose or use of affected aspect Perle v Meher (NY Ct. Ap. 2011) a rule requiring contemporaneous numerical measurements of range of motion could have a perverse results. Potential Ps should not be

70

penalized for failing to seek out, immediately after being injured, a Dr. who knows how to create the right kind of litigation NJ Serious Impact Interlude: P must show serious impact on daily life by objective evidence

NJ Model Civil Jury Charge Non-Economic Damages Entitled to fair and reasonable damages for full extent of harm, no more and no less Fair and reasonable compensation for any injury, temporary or permanent Disability or Impairment includes inability to pursue normal pleasure and enjoyment Life expectancy may be considered according to actuarial tables, but jury can adjust according to good judgment o Medical Expenses Account for reasonable costs, recovery limited to medical necessities o Loss of Profits Value of business lost, consider nature of business, average before and after profits (bring in expert economist) Present Discounted Value (Time Value of Money) o Discounted according to expert economic testimony o Calculate based on average interest and inflation rates for the past 30-50 years. o o Most of the time, the inflation and the interest rates cancel each other out (total offset method).

Juries instructed to calculate based on take-home after tax pay, since awards are not taxed (makes plaintiff whole as if injury never occurred) The lump sum rule o o o Ask the jury to award damages in a single payment. Now, there can be breakdowns in certain elements of the damages Present discounted value (the time value of money)

o Youre entitled to full compensation, not a windfall Structured settlement annuity retained earnings are not taxable if they finance a periodic payment of compensatory damages for personal injury. Its just the medium through which you are receiving your compensatory damages.

71

Taxable v. Non-taxable o Non-Taxable: compensatory damages arising from personal physical injury, including wage loss, emotional distress, disability and impairment, medical expenses, workers compensation. But youre not allowed a windfall, which is why in NJ you get the net wage loss and in NY the judge calculates the net after the jury decides on gross. o Taxable: interest on awards, compensation for lost wages or lost profits in cases other than personal injury, amounts received in settlements of pension rights (if you did not contribute to the plan), back pay and damages for emotional distress in Title VII civil rights claims, claims for business losses (e.g. copyright infringement, legal malpractice), punitive damages Liens Medicare, Medicaid, health & welfare funds, workers compensation, welfare and other public assistance (these first four are debts owed to the state), health insurers (subrogation generally barred) Wage loss Past and future lost wages. Past means from the time of the injury up to and including the day of trial. Plaintiff has a right to be compensated for any earnings lost as a result of injuries lost as a result of injuries caused by defendants wrongdoing. Embedded in as a result is the whole proximate cause framework.

New York does the gross and the judge does the calculation. New Jersey is based on the net or take-home pay. Only the take-home paythe amount left after taking out taxeswould have been available to plaintiff, and the amount you award is not subject to Federal and NJ state income taxes.

Madeira v. Affordable Housing Foundation (2d Circuit, 2006) Undocumented alien injured on job. Trial court instructed jury that they could not consider immigration status when determining liability but that they could when assessing damages for lost wages (alien status not relevant to other damage, e.g. medical costs). Rule: Federal immigration law did not preempt payment of lost wages for injured undocumented worker because it was the employer not the worker who violated the law.

72

Jury calculation incorporated the worker's removeability in deciding what, if any, lost earnings to compensate. The NY Labor Statute under which P is recovering was specifically intended to protect all workers; not just those of certain immigration status.

B. Caps, Remittitur, Additur, Collateral Sources


Caps on Damages (constitutionally vulnerable, see Arrington) o Right of citizens to adequate compensation v. costs transferred to rate-payers o Imposition of small cost on all citizens in exchange for imposition of large cost on injured parties The Harvard/Rand Factsheet The Inequitable Impact of NonEconomic Damage Caps: Three Academic Studies Demonstrate Severely Handicapped and Female Patients Are Hurt the Most Arrington v. ER Physicians (Louisiana, 2006) o Rule: The cap is unconstitutional because, per the LA constitution, it fails to provide adequate remedy. It does not allow the plaintiffs to recover fully for even their economic damages, much less all damages. o Concurrence: The cap is also unconstitutional on equal protection grounds (i.e. those with damages in excess of the cap are not treated equally to those who suffer damages below the cap and recovery fully). o The law is arbitrary and unreasonable: If you want to reduce insurance premiums, a cap on damages wont do it (as the historical record demonstrates); rather, you must deter negligent, tortious behavior. And studies show that doctors do not migrate between states on the basis of insurance premium rates. More generally, if the law does not accomplish its goal, and is unlikely to, then it is arbitrary. o Dissent: This statute may have been unconstitutional as applied to this Plaintiff, but that does not mean that it is per se unconstitutional. There are other damages caps (e.g. workers compensation, the states tort liability) that are not unconstitutional. Since there is no fundamental right to sue for malpractice and recovery full damages, then the proper review of the statute is rational basis review. There is a rational basis between the desire to maintain lower costs for medical malpractice insurance (and, therefore, healthcare costs for the citizens of the state) and a cap on damages. The burden of proof on the plaintiff then becomes to show that there is no rational basis between the law and the outcome desired by the legislature. If the law discriminates by creating different classes of people based on physical condition (as this law purportedly

73

does), then the proper review of the statute is reasonable basis. Charitable immunity (partial/whole) o o Varies from state to state (e.g. NY does not have, while NJ does have) They have a distorting effect on the litigation (your view of the facts is inevitably shaped by the interest of your client. E.g. decided to sue the doctor instead of the hospital cuz the hospital liability is capped all of a sudden makes the doctor more liable according to the way you will present the facts

Limits on recovering for intentional torts (via insurance) as a result attorneys will tend to veer away from arguing intentional torts because as a policy, insurance cannot be used to pay for punitive damages from intentionally inflicted torts. Even if your client wants to get the defendant and make him blameworthy, they might be better off arguing negligence. Limits on government liability: o o No prejudgment interest Credit for payments by collateral sources

The American Rule on counsel fees o Potential conflict of interest between you and your client (e.g. whether you get 25% v. 33%)

Litigation costs Collateral Source Rule traditionally payment of costs by insurance is not admissible as evidence in considering damages o Some states have legislated considering this in the damages section (but not for liability determination). Can recover full damages from tortfeasor even though you have already recovered rule of exclusion (often changed by statute) o Would be windfall to tortfeasor, victim should not be punished for foresight to buy insurance o NJ evidence of CSs meant to be excluded from jury, but insurance companies should be paid back o NY CPLR Sect. 4545 if court finds loss will be indemnified, the court will reduce award o NY puts number before jury, NJ does not o Exceptions to the Exclusion Rule (1) No-fault benefits paid (Not allowed in NJ, allowed but reduced in NY), (2) Union Welfare Funds, (3) Workers Comp. (NJ allows 100% reimbursement by employer), (4) FELA health ins. payments must be repaid, (5) Fed. Tort Cl. Act US doesnt repay liens, (5) Medicare if 3rd party

74

found responsible, must be repaid, (6) Medicaid NJ reduced lien proportionally Avoidable Consequences o A cause in fact inquiry a defendant is liable only for that portion of the injuries attributable to the defendants negligence Reasonably attributable to? Fairly attributable to? (embed these adjectives in there) You get to make a judgment on whether the connection is of a sort that it makes it fair to impose liability on another because even if you start a chain of events, everyone is an autonomous body (unless you are a ward of another). The plaintiff is not a billiard ball. He is himself an actor responsible to himself. The injury attributable to the plaintiffs unreasonable conduct is attributable to himself.

Example: Duty to mitigate damages (e.g. by medical and surgical treatment) Really, this is just a duty to take reasonable care of oneself (easier way to think about it than mitigating damages). You should then determine whether any of the plaintiffs injuries could have been avoided or alleviated by plaintiffs exercise of reasonable care to protect his/her own health. . . . Damages that could have been prevented by the plaintiffs exercising reasonable care are not the responsibility of the defendant. The tort system does NOT create a duty to repair. You should take necessary action to bring yourself back to functioning (e.g. repair your boat), and the judgment you receive gives you a right to the assets of the defendants to make you whole.

The burden of proof remains on the plaintiff, BUT as a procedural matter: The defendant must prove by a preponderance of the evidence that after its negligence occurred, the plaintiff acted unreasonably by either failing to seek or to submit to medical treatment in order to avoid further injury.

NY Pattern Jury Instruction:

75

1. What is the total loss suffered by the plaintiff regardless of fault? 2. Then you determine what the comparative negligence was regarding the initial accident and subtract the percentage attributable to the plaintiff. 3. Then you determine if the plaintiff failed to mitigate the damages and subtract for the proportion the plaintiff suffered as a failure to mitigate damages.

C. Wrongful Death & Survival Actions


Common Law youre worth more alive than dead o Three rules barred wrongful death actions: If the tortfeasor died, the claim against him did not survive If the tort victim died, the claim died (if it was for personal injury) Victims survivors had no claim of their own against the tortfeasor for loss of support or for emotional loss

Perverse incentives: you were better off killing someone than gravely wounding them.

Wrongful death and survival actions are statutory. Since theyre statutory, their parameters will vary by statute. Law of the place where the accident happened.

Wrongful Death Action o Is claim of survivors for their injuries. o Minority pecuniary losses only (e.g. New York, New Jersey) o o o o Majority emotional loss (or loss of society) allowed (most states) NY allows recovery for loss of services, parental care and advice but NOT for emotional distress and loss of consortium NJ allows monetization of loss of companionship, but not for emotional anguish As a practical matter, pecuniary losses used to mean that a death of a child earned you nothing (children were liabilities, not assets). New Jersey broke this down with a landmark case that essentially worked around the legislative restriction against emotional loss. So, the child was calculated to have likely provided care for the parents in the future during their old age. You start with the chores they perform around the house and continue until the parents old age. These things

76

are managed by expert testimony that has to be calculated by the market measure of the labor. Obviously, there is a lot of speculation in this but the basic goal of the tort system is compensation so were willing to adapt. o Life insurance is an inadmissible collateral source. Survival Action o Allows estate to sue for damages during period between injury and death (allows estate to recover all damages decedent would have recovered) o This is the cause of action that belonged to the deceased during her life (all personal injury damages incurred during life of the deceasedstops at death, e.g. no future wage loss or medical expenses). o The action survives her death and passes to her estate. The administrator brings the action on behalf of the estate (either the executor in the will, or someone appointed by the probate court deceased dies in intestate). o Recovery of damages that the deceased could have obtained before death o Typically allows for recovery of pain and suffering by the decedent o Compare loss-of-life damages with loss of enjoyment of life damages (loss of value decedent would have placed on his or her own life).

77

78

You might also like